Этого треда уже нет.
Это копия, сохраненная 27 апреля 2021 года.

Скачать тред: только с превью, с превью и прикрепленными файлами.
Второй вариант может долго скачиваться. Файлы будут только в живых или недавно утонувших тредах. Подробнее

Если вам полезен архив М.Двача, пожертвуйте на оплату сервера.
довен.jpg40 Кб, 512x337
Тупых вопросов тред 512263 В конец треда | Веб
Тред вопросов тупых.

Предыдущий: >>510503 (OP)
2 512266
С научной точки зрения почему оп - хуй?
3 512267
>>512262 →
И почему же?
4 512268
>>12266
Томущо он не пизда.
1024px-Sphericalfunctions.svg.png76 Кб, 1024x717
5 512287
Можно же электрон в атоме представлять сферической гармоникой? То есть если для атома нарисовать гармоники, соответствующие электронам на внешних незаполненных уровнях, то мы получим адекватную визуализацию этого атома?
6 512288
>>12287
Чел. Это волновая функция она не обладает физическим смыслом. Тебе нужен квадрат модуля.
7 512289
>>12288
Ничё не понял, какого модуля? Волновая — это ради бога, я хочу нарисовать атомы так, чтобы это соответствовало хоть чему-то, хотя бы какой-то математической абстракции.
Молекулы в электонный микроскоп.jpg135 Кб, 1156x839
8 512290
>>12289
Загугли "электронная плотность. атомы".
9 512291
>>12289
только проблема в том, что у тебя такое "облочное" представление, де-факто, 4хмерный график, который трудно нарисовать и трудно воспринять. возьми лучше с википедии схемы для орбиталей и успокойся
10 512320
>>12267
Например, потому что они обсуждали такую хуйню уже в то время.
11 512398
Каким образом наследственность влияет на поведение человека? Понятно, что есть гены, но как конкретно они влияют на когнитивные способности, поведение и тд?
12 512399
>>12398
А с чего ты взял, что она влияет?
13 512436
>>12399
А разве не влияет?
14 512465
Забавно что "направленные" процессы могут без проблем порождать ненаправленные процессы-например ток в проводах приводит молекулы и атомы провода в случайное движение во все стороны.

Но случайные процессы наверное практически никогда не могут приводить к направленным процессам...

Хм. хотя... есть же металлы, которые при нагреве выделяют ток? Как это работает?

Там стыке металлов какая-то типа мембрана получается односторонняя, которая при случайном колебании туда-сюда электронов от нагрева пропускает их только в одном направлении?
15 512485
>>12465

>ток в проводах приводит молекулы и атомы провода в случайное движение во все стороны


Что ты имел ввиду то блять?
2020-07-3116-52-14.mp41,2 Мб, mp4,
1620x540, 0:05
16 512490
>>12287
Пока вот такая получилась
17 512491
>>12465

>есть же металлы, которые при нагреве выделяют ток? Как это работает?


Ты имеешь ввиду термоэлектрический эффект?
Почитай хотя бы на википедии как он работает.
18 512492
>>12490
Падажи. На мпешке визуализация обитателей водородоподобных атомов. Многоэлектронные атомы же взамино пидорасят друг друга орбитали в итоге там картина будет совсем другой.
19 512493
>>12492
Ага, есть такое. Так что это визуализация не атомов, а сферических гармоник электронов на внешних уровнях и незаполненных орбиталях. Они ещё и не у всех посчитаны правильно, ворк ин прогресс
20 512502
Вопрос по механике. Пусть у нас есть шарик, который катится по ровной поверхности. Из-за трения он замедляется. Чтобы сохранялось качение, скорость его вращения должна оставаться пропорциональна поступательной скорости. Так вот, как именно это происходит? Трение качения (я знаю, что это "фиктивная сила", по факту там реакция деформированной опоры так влияет) уменьшает одновременно импульс и момент импульса?

А если перераспределить внутренности шарика, уменьшив или увеличив его момент инерции при той же массе? Ведь тогда коэффициент пропорциональности должен тоже стать другим.
Untitled.png92 Кб, 1872x619
21 512504
>>12502
Если тебе надо решить домашку, то считай через закон сохранения энергии, а не через импульс. Если шарик в итоге останавливается, значит его суммарная кинетическая энергия полностью перешла в работу силы трения. Из пройденного пути, массы шарика и коэффициента трения легко определить эту работу.
22 512505
>>12502
Тут же даже никаких лангражеанов не надо. Просто запиши уравнения Ньютона.. Пиздец ты.
23 512506
>>12502
Ах да, скорость центра масс всегда равна произведению угловой скорости и радиуса, не важно шарик это, колесо или цилиндр, не важно полый или сплошной (моменты инерции у них разные), иначе тело катиться не будет.
24 512507
>>12287
У тебя на рисунке гармоники и нарисованы собсна, не понимаю вопроса.

>>12291
Какой 4-ёх мерный, долбоеб? Говна от ударитьмусор обкурился?
25 512508
>>12507

>не понимаю вопроса


>Какой 4-ёх мерный, долбоеб?


>не понимаю


Оно и видно, школотун
26 512509
>>12507
Вопрос был адекватно ли представлять атом как набор этих гармоник
27 512510
>>12509
Нет, не адекватно
28 512513
>>12485
Нагрев от сопротивления электрического.
29 512514
А вот металлы ток проводят потому что в них свободные электроны типа есть, так выходит?

Но почему они не вылетают из металла высоком напряжении? Что их там держит?
А если их что-то так хорошо держит, то почему они так легко течь начинают? Даже при низкой разности потенциалов

И если там в металее свободные электроны типа... но метал же электронейтрален если на него никак не воздействовать.
30 512515
>>12514
Электроны относительно легко перемещаются внутри металла, образуя что-то вроде электронного газа
Наружу при большом напряжении они вылетать могут, нули катодные трубки
Держит их внутри положительно заряженные ядра атомов
31 512516
>>12514
Кроме свободных электронов там есть еще кристаллическая решетка, в узлах которых положительно заряженные атомы металла, они и удерживают электроны. Электроны могут свободно течь внутри решетки между узлами, но не покидать ее.
32 512517
>>12263 (OP)
Я не понимаю практическое применение многих научных физических работ.
Вот почитал я серьезный научный труд в серьезном журнале о сфере дайсона вокруг черной дыры, где радостные ученые вычислили способ, метод и математическое обоснование как получать таким образом энергию. Статья завершалась восторженным "осталось лишь дождаться появления таких материалов!". На сопр материалов кстати все и держится и возможно таких материалов и не будет никогда.

То есть группа ученых несколько лет проводила огромный блок вычислений для математического обоснования использования плохо изученной аномалии.

НА-ХУ-Я? Я не понимаю смысл такой науки и зачем ее поощрять. Это ведь банальная задачка на математику, причем не самая сложная просто объемная по вычислениям. Они ведь даже теорий новые не сделали.
Почему это считается наукой?
15916918959201.jpg26 Кб, 420x423
33 512518
Квантуется ли как-то пространство? Если нет, то любое растояние можно бесконечно дробить на всё меньшие отрезки - и как тогда происходит движение, если между делениями сантиметровой линейки - бесконечность?
34 512519
>>12517
Лучше спроси нахуя нужна манятеория эволюции? А психология? Или джендер-стадис? На что выделяют деньги- то и исследуют. Все топовые универы выделяют гигантские бюджеты на такие вот "исследования". Представь себе, какая-нибудь жирная ебанашка с крашеными волосами, целыми днями сидит и выдумывает откровенный бред и за это получает зарплату как у профессора физики/химии/математики.
А все потому что деньги и власть в этом мире находятся в руках дьявола, который развращает быдло всякой хуйней вроде скайфая, феминисток, пидарасов и трансгендеров. Ненавидь мир, люби Христа.
36 512521
>>12517

>НА-ХУ-Я


Большая часть науки не имеет на это ответ. Нахуя нужно СТО, нахуя нужно ОТО, нужен БАК - лучший ответ это ХЗ. А 100 лет газад был вопрос "нахуя исследовать полупроводники". Это все может в будущем дать убер-плюшки по типу ядерного реактора слияния на мюонах. А может нихуя не дать, как теории обратимых вычислений. А как тогда выбирать, какая наука достойна выдеоения финансов, а какая нет? Сейчас делают по уровню заинтерисованности в области научного сообщества. Если чел горит желанием слелать сложную хуйню максимально хорошо, то если она пригодится это будет вмн, а если нет, то он чего-нибудь научится и это тоже вин, но маленький.
37 512522
>>12520
Так эксперименты вроде не зафиксировали зернистости пространства?
38 512523
>>12517
>>12519
https://www.chapman.edu/our-faculty/amy-moors

> Assistant Professor


> Much of her research focuses on issues related to gender, sexuality, diversity, and well-being


Последняя статья:

> Moving past the rose-tinted lens of monogamy



https://www.chapman.edu/our-faculty/files/curriculum-vita/amy-moors-cv.pdf

> Research Funding and Grants


> $1,650,000



Напоминаю что психология считается наукой. Доказывать быдлу что ебаться с кем попало это полезно и правильно это наука. Доказывать быдлу что быть куколдом это признак высокого интеллекта это наука. Доказывать быдлу что отдавать малолетнего ребенка на гормональную терапию по смене пола это наука. И бабло на это выделяют колоссальное.
39 512524
>>12522
Прочитай ссылку и все поймешь:
Heisenberg uncertainty principle. This general principle of the universe states that it is impossible to measure position and momentum simultaneously with infinite precision — measure one well and the other will be measured poorly.
image.png2,6 Мб, 1400x934
40 512525
>>12523
Да, психология это наука, а наука это попил, развод и наебалово. Что сказать-то хотел?
41 512528
>>12508
Окей, по факту будет что сказать?)

>>12509
Да, адекватно.

>>12510
С хуев ли?
42 512530
>>12528
Во-первых это волновые функции а не электронная плотность. Во-вторых, это даже не точное решение а лишь приближение, где мы закрываем глаза на корреляцию электронов.
43 512533
>>12530
Так каждой гармонике соответствует свое облако со своими числами заполнения, что собственно и есть то что нужно опу. Пытаться нарисовать электроны как физические объекты в атоме это глупость.

ну так что там с четырехмерным графиками, клоун?
44 512534
>>12504
Мне не нужно решать домашку. Я хочу понять, каким образом энергия поступательного движения и энергия вращения уменьшаются согласованно, хотя коэффициент пропорциональности между ними может быть разный, в зависимости от внутренностей шарика.
45 512535
>>12534
Я в следующем посте уже разжевал >>12506
Повторю ещё раз, условие качения без проскальзывания:
v линейная скорость центра масс = угловая скорость ω * r радиус
Оно действительно для любых катящихся тел и не зависит от момента инерции.
https://v-kosmose.com/fizika/kachenie-bez-skolzheniya/
46 512536
>>12533
про четырехмерный график писал я. не этот анон.
облако это не 2хмерная поверхность вокруг атома, оно имеет некоторую "размытость". Т.е если ты плоскостью просечешь это облако, то ты получишь, что это не тонкая линия, а такое вот пятно, которое имеет пик плотности на линии, который плавно спадает при уходе с неё. это можно представить как трехмерный график. только проблема в том, что в случае р орбиталей одной плоскостью уже не обойдешься, нужно строить в трехмерье, а график, который каждой точке из R3 сопоставляет число 4хмерный.
47 512537
>>12535

>Повторю ещё раз, условие качения без проскальзывания:


>v линейная скорость центра масс = угловая скорость ω * r радиус


Бля, я понимаю. Почему это условие выполняется? Каким образом импульс движения катящегося тела остается пропорционален моменту импульса, если один зависит от массы, а другой от момента инерции? Почему одна и та же замедляющая сила по-разному меняет эти две величины для шариков, одинаковых внешне, но имеющих разное распределение массы внутри?
48 512538
>>12537
Зачем ты выдумываешь какие-то импульсы? Связаны скорости, по ссылке разжевано как и почему.
49 512539
>>12538
Ясно.
50 512540
>>12536
Чего блядь? То что ты описал - это трехмерная фигура.

Волновая функция в данном случае зависит от трёх переменных, радиуса и двух углов, откуда там четвертая переменная то?
51 512546
Прочитал про эксперимент Бенджамина Либета. Как жить дальше нахуй?
52 512547
В игре Биошок Инфинити мне понравилась такая фишка, что Соловей (киборг-слуга) управлялся музыкой, определённая комбинация нот заставляла его выполнять определённую команду, тогда я подумал, что это просто фантазия разработчиков.
А недавно посмотрел Прометея, и там тоже есть момент, где в качестве пароля использована комбинация нот.
Это што, какой-то новый тренд в кибербезопасности, или-таки просто элемент научной фантастики? Ничего не нагуглил, мб как-то не так искал
53 512548
>>12546

>Как жить дальше нахуй?


Понять, что так называемое "решение" принимается ранее, чем ты его осознаешь. При этом оно, условно, остаётся решением.
54 512549
>>12548

>"решение" принимается ранее, чем ты его осознаешь


Но ведь тогда получается, что мной кто-то управляет!
55 512550
>>12540
какая нахуй четвертая переменная? три переменных и одир результат. для того, чтобы построить график нужно 4хмерное пространство.
56 512551
>>12549
Ты сам и управляешь. В контексте принятия решений не имеет смысла отделять сознание от подсознания, ибо человек, даже с наличием такого уникального инструмента, как сознание, всё равно остаётся животным.
57 512553
>>12540
>>12550
Два еблана не могут разобраться с понятиями из-за отсутствия в их запасе слов связки "эволюция системы во времени"?
58 512555
>>12550
Для визуализации рисуют области, где квадрат модуля принимает ненулевые значения читай: границу области. Анону и нужна визуализация, а не пиздец подробное распределение. да и то, его цветом можно изобразить
59 512556
>>12553
Это стационарная задача, дурик.
60 512558
>>12555

>место, где квадрат ненулевой


мб максимум, он во всем пространстве ненулевым будет.
если максимум, то это как раз и получатся орбитали.вроде
61 512559
Нужно сделать детали из epdm резины. Что для этого нужно? Где почитать о способе производства из такой резины?
62 512560
>>12533

>каждой гармонике соответствует свое облако со своими числами заполнения


Только в маняфантазиях с приближенным решением. Электронные плотности никаких гармоник не имеют. Кончай смешивать понятия. Анон хочет изображение атомов. Ближайшая аналогия к изображению атомов это электронная плотность. Волновая функция не является физически-наблюдаемой величиной. Эти орбитали вообще маняфантазия на постном масле. Волновая функция электронов атома гелия, в котором два электрона, зависит от шести пространственных координат и двух спиновых.
2020-08-0119-21-40.mp42,1 Мб, mp4,
1620x540, 0:05
63 512561
минус модуль плюс цвета
64 512566
>>12560

> >зависит от шести пространственных координат


Это ты в каком пространстве шесть координат высрал?
65 512567
>>12566
в пространстве x1 y1 z1 s1 x2 y2 z2 s2
66 512568
>>12567
А, бля, таки это координаты двух электронов
А ты, я так понимаю, хочешь, чтобы атом, содержащий, два электрона, описывался бы без привлечения координат этих электронов?
67 512569
>>12558
Не, орбиталь - эта часть пространства, в которой вероятность найти электрон 90%
68 512577
>>12568
Ему нужно изображение а не волновая функция.

>А, бля, таки это координаты двух электронов


Да, но проблема в том, что они все входят в функцию. Изобразить это можно только в шестимерном пространстве более менее адекватно. Орбитали же, это такой маневр жопой, что мы будто бы закрываем глаза, и говорим, ну может же оно не F(x1 y1 z1 s1 x2 y2 z2 s2) а F(x1 y1 z1 s1)хF(x2 y2 z2 s2). Оно конечно похоже получается, но часть решения теряется, видно по энергии, которая не совпадает с экспериментальной.
69 512578
>>12577
Ну, тогда уже в семимерном, получается
А так, ты же ещё пропустил приближение, согласно которому рассматривают отдельно атомы и электроны
Так то да, там при практическом решении приближений дохуя, но в итоге цифры более-менее сходятся с экспериментом особенно, когда берут какие-то цифры из эксперимента
70 512579
>>12577
Ящетаю, что нет ничего криминально в том, чтобы считать электрон простым осциллятором для целей нарисовать картинку, по которой будет видно отличие одного атома от другого, F(x1 y1 z1 s1 x2 y2 z2 s2) норм тема.
71 512582
72 512588
>>12578
если выебываться, то учти еще, что спин это двумерная величина
73 512593
>>12588
Ну, бля, собсно, по этому и нормальной теории, по которой модно было бы точно посчитать молекулы (да даже тот же гелий) нихуя нет
74 512599
>>12518
Есть расстояние, есть скорость-так и передвигается.

Ты сам обсираешься в вопросе, раз зернистости не зафиксированно, то и проблем в перемещении нет.
Вот если бы была зернистость, дохуя мелкая и много, то пока переберёшь всю её...
75 512600
>>12593
Всмысле? Всё есть, просто там рассчёты пиздец получаются. Даже одну молекулу.
76 512605
Существует ли подтвержденная возможность создания светопроницаемых силовых полей, блокирующих твердую материю как минимум? Ну, как в кино.
77 512611
>>12605
Нет, не существует.

https://ru.wikipedia.org/wiki/Фундаментальные_взаимодействия
Сильное/слабое вообще игнорь, в макромире их нет. Электромагнитное взаимодействие работает только с заряженными частицами. Ну а про гравитацию никто ничего толком не знает: фшизики выдумывают мозгодробительные уравнения, одно охуительнее другого, но найти практическое применение даже не пытаются, т.к. знают что все это попил и пиздеж. Особо борзые фшизики вообще пятую силу придумали https://ru.wikipedia.org/wiki/Пятая_сила кто знает может завтра и шестую найдут, а воз и ныне там.
78 512612
>>12611
У тебя /zog/ протёк.
79 512613
>>12600
Точных нет, там используется три приближения подряд (приближение Борна-Опенгеймера, рассматривание валентных электронов отдельно и одно электронное приближение)
И после этого эту херню уже считают разными методами, и расчёты с экспериментом расходятся часто
правда, другие модели вообще нихуя не дают, но это уже другая история
80 512614
>>12556
Гамильтониан изо рта вытащи и в уравнение своё СТАЦИОНАРНОЕ подставь, идиот
81 512616
хватит ли энергии при сонолюминесценции для инициации химических реакций, то есть для разрыва химических связей?
82 512618
>>12611

>Электромагнитное взаимодействие работает только с заряженными частицами.


И ты такой проваливаешься сквозь стул, ибо у тебя жопа состоит из нейтральных атомов.
83 512619
>>12616
Можно.
84 512625
>>12619
а есть ли статьи на эту тему в научных журналах?
ну там про создание экспериментальных каталитических реакторов, например?
85 512626
>>12625
Не знаю, думаю что не очень много. Так как есть лазеры и всякое такое. Типа "нахуя"
86 512627
Знаю, что препараты, выводящие радиацию из человеческого тела, не доступны людям потому что будь иначе и люди бы не умирали от старения. Ну а чисто в теории, возможно ли вывести радиацию из клеток с помощью употребления веществ, которые эту радиационную хуйню поглощали бы и выводили из всех тканей тела, или всё возможное в этой области уже изучено и перепробовано?
87 512628
>>12618
пиздуй учить школьную физику. раздел "строение атома"
88 512630
>>12628
Если ты не можешь представить вид потенциала, необходимого, чтобы атом не мог свободно пролететь через него. То у меня для тебя новости
89 512632
>>12627
Что ты имеешь ввиду под "выводить радиацию"? Выводить радиоактивные вещества? Ну таки от радиоактивного калия, например, ты не избавишься никак.
Та и ты считаешь, что старение из-за радиации происходит? Ну тогда подумай о том, что вокруг нас естественный фон, от которого тоже не избавиться
90 512636
>>12632
Теперь я подумал об этом и самому стало очевидно. Чтобы существовать дольше нужно иметь более совершенный бешено регенящий без ограничений организм, либо жить под "твердью небесной" из защитной водно-парового слоя атмосферы.
91 512637
>>12636
Та нихуя, там же при делении клеток постоянно случайные ошибки вылезают
К тому же, от естественной радиации ты вообще хуй избавишься
92 512639
>>12263 (OP)
Напомните как называется парадокс теории игр(?), суть его раскрывалась на примере выбора дорог без пробок и на примере выбора рандомного числа от 1 до 50, когда побеждает тот, кто выберет наиболее близкое к среднему арифметическому выбранных числе.

Например, 10 человек спросить выбрать рандомное число между 1 и 50. Первым шагом глубины(?) будет выбрать число 25. НО - вы можете подумать, что такое число выберет большинство, поэтому выбираете 24, но потмоу вы думаете что так как вы поудмают и другие, и вы выбираете 26, но тут вы думате, что ИОНИ выберут 26 и т.д. и т.п.
93 512645
>>12639
Ориентируясь на правила собственного мушления я бы предположил, что мало бы кто подумал банально поделить 50 пополам. Самыми популярными бы были числа "36", "13", "50", "12" и "49" по убывающей. Но я бы поставил на "42", если бы знал, что множество опрошенных это офисный планктон среднего умишки в возрастном диапазоне 25-45 лет. А че за игра не знаю.

мимо
изображение.png5 Кб, 299x430
94 512653
Может ли находящийся в сознании человек каким-то образом сделать так, чтобы оптические оси глаз пересеклись не спереди, а сзади?
95 512659
>>12653

>каким-то образом


Изи, вытащив глаз.
96 512661
>>12630
а ты можешь представить атом, состоящий нейтральных частиц, чтоб он с полем не взаимодействовал?
97 512662
>>12653
будучи косоглазым он всегда в таком состоянии)
98 512669
Привет, аноны. Обращаюсь к знатокам японской действительности. Нужны относительно недавние материалы, статьи, книги, по вопросу тёмной стороны технологий, виртуального, роботического прогресса, с реальными фактами и т.п.
Буду благодарен, если вы поделитесь.
99 512677
Сап научный. У меня пара вопросов по теме треда: какого размера и на каком расстоянии должен быть объект, чтобы при наблюдении с Земли закрывать все небо хотя бы из одной точки наблюдения? Ну и расстояние не должно быть близким, не ближе стратосферы.
100 512678
Поясните почему ридберговские атомы раздутые вплоть до мм не становятся видимыми?
101 512680
>>12678
Потому что его раздутость выражается в увеличении радиуса орбиты внешнего электрона, а не диаметра самих электронов или ядра лол.
102 512684
>>12680
Ток орбитали, а не орбиты
103 512693
>>12680
Непонятно, а в обычном веществе не электроны отражают свет?
104 512695
План для ГЭС такой. Строим на глубине (метров 500) здание с турбиной, давление воды приводит в движение турбину, мы получаем электричество и считаем шекели. Где я проебался?
105 512696
>>12695
А куда ты воду из глубины девать будешь? И как закачивать всё новую наверх здания?
106 512697
>>12693
Электроны, но каждый примерно одинаковый электрон отражает (на самом деле, конечно, не отражает, но не суть) одинаковое количество света
107 512698
>>12696
Хитрый ход - вода падает в пропасть и крутит турбину, на дне пропасти лава, вода превращается в пар и поднимаясь крутит вторую турбину!
108 512699
>>12698
Ну, для начала - примерно это используется в Исландии
И тогда проще крутить турбину уже паром, он ж большее давление создаёт
Ну, и остаётся вопрос, как ты собираешься строить турбину рядом с лавой нахуй, да ещё и на такой глубине (и да, лава у тебя будет не на 500м, а примерно на 30 км)
109 512701
>>12696
Мы построим трубу, ведущую на поверхность, вода сама будет выливаться. Это же вечный двигатель!
110 512702
>>12697
Я так и не понял в чем будет разница в плане отражения между множеством электронных оболочек кучки атомов, которую можно увидеть, и одним раздутым до такого же размера атомом.
111 512703
>>12702
Смари, когда у нас есть куча атомов (а в веществе их просто дохуя), каждый атом отражает один квант света (не важно как), и в сумме получается дохуя света
Если же у тебя один атом, то он отражает только один фотон, и хуй ты его сможешь увидеть
112 512704
>>12701
Ну, это конечно не вечный двигатель, ты просто превращаешь в электричество тепло земли
Но таки именно так Исландия и живёт, правда там эти трубы природные
113 512705
>>12703
В целом, понятно. Но недавно была новость, что камерой с большой выдержкой и фильтрами удалось заснять один большой атом в виде точки светящейся.
123.png31 Кб, 1662x618
114 512706
>>12704
Ну все, значит вот проект технологичного города будущего на воде, экологически чистого и автономного. Где тут запантетовать мой проект?
115 512709
>>12706
А теперь скажи, чем твоя хуйня, требующая хуй знает каких затрат, лучше, чем АЭС, например?
116 512710
>>12709

>лучше, чем АЭС


1. Не требует огромных затрат на обслуживание.
2. Нет опасности радиации.
3. Технологически проще реализовать.
117 512712
>>12710
1) ты ебанат? Ты думаешь, трубу загрузил, и с ней нихуя не будет? Там же дикие температуры и давления
2) ну, есть опасность взрывов
3) ну хуй знает, одну только скважину сверлить пиздец как сложно
118 512714
>>12695
>>12706
Чтобы турбина двигалась тебе нужно как-то создавать пониженое давление. Хуйня на картинке работать не будет, так как в обоих трубах будет одинаковое давление столба воды. Чтобы работало уровень воды в требе где выливается должен быть ниже - но тогда у тебя обычная ГЕС.
119 512715
>>12547
Это просто художественный прием. Практической пользы нет. Технически проблем сделать такое нет, если поебаться.
120 512720
Есть ли погрешность в 3 законах Ньютона? Ну например, если мы говорим, что I=U/R, мы пренебрегаем, что проводник из-за протекания тока может нагреться, а при нагревании изменится R. А если мы говорим |F1|=|F2| мы чем-нибудь пренебрегаем?
121 512722
>>12720
Какие нахуй погрешности? Законы на то законы, что они являются чем-то абсолютным в рамках теории. Более того, физические законы формулируются в дифференциальном виде, а это значит что связываются мгновенные величины. Даже если сопротивление флуктуирует, его мгновенные величина все равно связаны с током и напряжением строгим образом.

А все 3 закона Ньютона сводятся к более общим соображением - принцип стационарного действия и однородности/изотропности пространства. Законы будут строго выполнятся если у нас в пространстве нет никаких магических хуевин.

В релятивистской теории же законы Ньютона не используют, вместо них используют те же общие соображения, с разницей в том, что рабочее пространству имеют другу структуру.
15732137310140.jpg346 Кб, 850x677
122 512723
Сильный ИИ хоть теоретически возможен?
Зачем мне это: передать эстафету разума, сбросить бремя сознания и самоосознания и спуститься по лестнице Иакова к блаженству животных.
123 512724
>>12723

>Зачем мне это


Не ты это решаешь.

>эстафету разума, сбросить бремя сознания


У тебя этого нет, ты просто кусок мяса, животное, набор алгоритмов.
124 512725
>>12723

>Сильный ИИ хоть теоретически возможен?


Нет. Программа (то есть непосредственно код) работает с синтаксисом, сознание (то есть человек) с семантикой.
125 512734
>>12701
Не построите.
По той же причине по которой никто не строит заводы по производству кроссовок роботами.
Эти заводы строят в регионах Южной Азии, где зарплата в месяц 40 евро.
Один абориген с комплектующими со склада произведет тебе вручную 10 пар кроссовок за месяц и возьмет за это 40 долларов.
Робототехнический завод мог бы производить 1 000 000 под управлением 40 человек. Но такая схема никогда не принесет 300% прибыли.
Поскольку ты живешь в эпоху развитого капитализма, то единственной движущей силой является увеличение прибыли любыми способами.
конкретно твою долгосрочную постройку просто порешает рыночек.
И вовсе не обязательно строить вкопанную трубу.
Можно построить высотой метров 300 с определенным дизайном поверхности вокруг, чтобы восходящие потоки воздуха вращали тебе турбины бесплатно.
Но тогда нефть, которая добывается по 10 не продастся по 50 долларов за баррель.
126 512735
>>12723

>Сильный ИИ хоть теоретически возможен?


Пока никто не опроверг эксперимент под названием "Китайская комната" - нет.
127 512736
>>12712

>Там же дикие температуры и давления


С трубами, которые сверлили на глубину 12 километров вроде до сих пор все в порядке.

>есть опасность взрывов


Каких взрывов? Нет таких энергий на глубине.
Там градиент по температуре что-то около 1 градуса на 100 метров. Исландия живет гейзерами, которые живут вулканической деятельностью.
>>12712

>одну только скважину сверлить пиздец как сложно


По сравнению с постройкой Ядерного Реактора?
Надеюсь, ты не серьезно.
15965373336583387303183883486416.jpg1,7 Мб, 2976x3968
128 512738
>>12720
да, пренебрегаем тем, что взаимодействие идет через поле, которое может нарушать это равенство. пример летящие вот так электроны. 1 на 2 действует, а 2 на 1 - нет.
129 512739
>>12735
А если и я сам эта комната?
130 512741
>>12738
Если вместо силы брать "4-силу", то всё нормально там.
131 512745
>>12738
>>12741
ОБъясните?
132 512750
Сап, саеноны, самый гениальный в мире вопрос:
Почему время есть 4 измерение? И представляют в нем проекции предметов как растянутые их копии от начала существования до их конца?
Тип, допустим, если у нас понизить мерность убирая одну - длину, ширину или высоту, то ведь время так и останется для флатландии, для 3д и для 4мерного мира универсальным?
В чем главная тупость вопроса?
133 512751
>>12750

>Почему время есть 4 измерение


Так удобно считать, всё.
134 512756
>>12723

> Сильный ИИ хоть теоретически возможен


На данный момент это упирается в философию. Тебе нужно определить свое отношение к двум предположениям:
1. Материализм верен, идеализм сосет.
2. Вселенная позноваема полностью (тоесть можно создать модель любого существующего физического процесса).
Если ты считаешь что на оба продположения стоит ответить "да" то сильный ИИ возможен.
135 512757
>>12750

>Почему время есть 4 измерение?


Не есть.
136 512758
>>12750

>В чем главная тупость вопроса?


В том что времени нет и не нужно.
137 512759
>>12758

>времени нет


>Аноним 04/08/20 Втр 18:09:05 №512758 136


>18:09:05

138 512760
Вообще считаю каким-то сюром, что магниты именно магнитят,а не создаю сопротивления движению, типа медленного опускания в их поле, и чем сильнее магнит тем медленнее.
Также и гироскоп, что он не просто медленно опускается, а прям конретно парит на одной точке опоры и с центром масс далеко за осью равновесия/не падения с этой опоры.
139 512761
Где лучше всего спросить насчёт вката в ноуку?
140 512762
>>12759
Ну и что за херню ты скинул?
141 512763
>>12762
Время твоего поста
142 512764
Речь это изобретение, открытие или эволюция?
image.png444 Кб, 600x600
143 512765
>>12764
Эволюция, причём задолго до появления приматов.
15791271940410.png777 Кб, 1017x1080
144 512766
Если я магическим образом создам абсолютную копию себя, то перенесусь во второе тело, или зависну в обоих телах сразу? А почему нет? Если копию создать, пока я в отрубе, то как я узнаю, что я не копия?
145 512767
>>12758
Тогда что есть?
146 512768
>>12766
Никакого "тебя" нет. Переносить нечего.
Когнитивное искажение приводит ко временной иллюзии и в первом, и во втором теле. Но, поскольку тела находятся в разных местах, эти иллюзии разные. Твоя абсолютная копия не будет копией, это будет равноправная версия тебя.
147 512773
>>12768
Это не я из параллельного Мира, это атомы, сложившиеся так, чтобы точно повторить моё тело. То есть копия. И копия примет это, потому что мне это известно. Цимес-то в том, что на мгновение у нас будут абсолютно одинаковые мозги, мозг на двоих так сказать, и фиг его знает, что с этого произойдет по правилам этого Мира.
148 512774
Тут есть раздел про билогию?
149 512775
>>12766
Soma - сыграй в неё
150 512776
>>12773
Есть теорема о запрете клонирования. Кванты запрещают.
151 512779
>>12776
Ну, они будут не в одной точке пространстве, так шо не запрещают
152 512785

>эксперимент с фотонами и двумя створками


>разные результаты при отсутствии и наличии наблюдателя


>камера считается за наблюдателя


>ИТОГ — БЫДЛО СЧИТАЕТ, ЧТО МИР ЭТО МАТРИЦА



Чегоблядь? Так, тогда во-первых: какого хуя бессознательная камера считается за наблюдателя? Почему следы на стенке не считаются за наблюдателя, если фактически они свидетель измененного состояния? Ведь если "мир — это матрицо", то программа должна была отследить, что мы все равно узнаем о её жульстве по этим следам, а следовательно за все кукареки на эту тему можно пердеть кукаретику в лицо. А об истинной причине такого положения дел никто еще даже не предположил получается.
153 512786
>>12785
Проиграл с твоего троллинга тупостью
1578701471770.jpg55 Кб, 640x491
154 512800
>>12263 (OP)
Можно ли после окончания магистратуры по "информатика и вычислительная техника" пойти в аспирантуру в НИИ и делать ёба-штуки? сейчас на специалитете по компьютерным сетям и программированию в них
155 512803
>>12800
Бампаю вопрос. Та же ситуация только я бакалавра закончил.
156 512806
>>12800
если пройдешь экзамены, то почему бы и нет?
157 512807
>>12800

>пойти в аспирантуру в НИИ и делать ёба-штуки?


что ты под этим подразумеваешь?
15963682643070.jpg182 Кб, 933x902
158 512809
>>12807
Я всегда хотел стать учёным и работать в НИИ, а не на дядю
159 512810
>>12263 (OP)
Строим огромное кольцо в космосе (диаметром в с Луну), внутри кольца разгоняем материю. Поскольку сопротивления воздуха нет, материя замедляться не будет, значит она будет двигаться и ускоряться бесконечно. Можно из этого как-то получить энергию?
160 512811
>>12809
Боюсь тебя разочаровать, но по началу ты будешь работать на научрука как на дядю. Особенно без опыта в науке, будешь делать ту хуйню, которую дяде-научруку влом делать - она долгая, нудная, без креативу, но и без риска, что ты в другую сторону начнёшь исследовать. Причём от этого дяди у тебя зарплата будет куда сильней зависеть, чем в других организациях - на обычной работе редко бывает, что премии превышают оклад в разы, а в науке с нищенским окладом ты только на премию-в-разы-выше-оклада и сможешь выжить. Если ещё найдёшь успешного и доброго дядю. Со временем, конечно, сам станешь "дядей", но с довольно долгим временем. Вот и думай.
161 512812
>>12810
Эх, опять попытка наебать термодинамику.

>ускоряться бесконечно


И даже 1 закон Ньютона.
162 512827
Чо за хиггсовское поле? Я просто нихуя не понимаю, как мне кажется. Вот есть элементарные частицы, это неделимые ребята, из них состоят составные частицы (а вся материя состоит из фермионов). А хиггсовское поле это что вообще? На википедии написано, что эта штука определяет инертную массу, то есть способность тела сопротивляться фундаментальному взаимодействию и состоит из бозонов хиггса.
Но я не понимаю как это должно выглядеть. У каждой элементарной частицы переносчика взаимодействия есть поле хиггса?
Я не понимаю что происходит с частицами-переносчиками, когда они исчезают. Это вообще как? Они же не могут просто исчезнуть!
163 512833
>>12827
Частицы это тебе не шарики какие-то.
164 512838
>>12811
Ну, тут ещё важно сказать, что этот дядя (если он нормальный) потихоньку обучает тебя
Из-за этого, кстати, научруки очень не любят, когда чел уходит в другую группу - в тебя вложили деньги и усилия, а ты сваливаешь
Но я слышал историю от чела, жившего в Америке 5 лет, что там некоторые научруки пиздец охуевшие - они, зная, что от них зависит, останется ли студент из чужой страны в Америке или нет, эксплуатировали этих самых студентов как только могли: заставляли работать ночам, или приходить несмотря на болезнь
165 512844
>>12827
Начнем с того, что такое поле.
Поле это гладкая функция от координат, т.е в каждой точке пространства-времени задано число, тензор(скаляр, вектор, н-формы), спинор (вектор модифицированный так, чтобы его можно было "умножать" на подобные себе, в некотором роде это вектор вращения, отсюда и название); гладкий значит что при бесконечно малом движение по координатам, объект так же будет бесконечно мало меняться. Часто поля наделяют особыми свойствами не изменяться при определенных преобразований объектов или координат, эти свойства называют симметриями. Из симметрии следуют важные следствия: симметрии можно соотнести некоторую постоянную величину(энергии, импульс, заряд) и неявность функции от координат(это дает некоторый произвол в выборе функций, но так же сильно ограничивает функции определенным классом). Помимо этого симметрии позволяют компактно и красиво записывать уравнения полей и даже строить другие поля исходя из симметрии.
Теперь к элементарным частицам.
Квант это порция, это значим когда мы детектим фотон или электрон, мы получаем не состояния частицы, а всего общего поля(для фотона это будет детектция ЭМ поля, а для электрона электронного поля).
По отдельности мы можем легко составить уравнения поля для ЭМ и электронов, однако если мы захотим состыковать их вместе в единое общее поле, то возникают ряд трудностей. Все дело в симметриях полей. Масса электрона оказывается "лишней" и она вписывается в уравнение движения как совершенно левый/внешний параметр, покушающийся на святое. Для ЭМ поля это не особо критично, все веселье начинается тогда, когда мы начинаем объединять электромагнитное и слабое взаимодействие. Из этого объединения строго следует, что все частицы должны двигаться со световым скоростями, а если мы захотим ввести массу, то лезут чудовищные расходимости.
И тут приходит к Хиггсовскому механизму. Суть в его том, что вводится дополнительное поле, которое нарушает симметрии электрослабого взаимодействия. Ну или другими словами хиггсовское поле взаимодействует с электрослабым полем так, что в уравнение поля возникает дополнительный член, соответствующий массе. Хиггсовское поле наделяет массой только слабое взаимодействие, остальные частицы(электроны/мюоны/кварки/а вот адроны уже отдельная история) уже приобретают свою массу просто из того, что они взаимодействует со слабым полем.
165 512844
>>12827
Начнем с того, что такое поле.
Поле это гладкая функция от координат, т.е в каждой точке пространства-времени задано число, тензор(скаляр, вектор, н-формы), спинор (вектор модифицированный так, чтобы его можно было "умножать" на подобные себе, в некотором роде это вектор вращения, отсюда и название); гладкий значит что при бесконечно малом движение по координатам, объект так же будет бесконечно мало меняться. Часто поля наделяют особыми свойствами не изменяться при определенных преобразований объектов или координат, эти свойства называют симметриями. Из симметрии следуют важные следствия: симметрии можно соотнести некоторую постоянную величину(энергии, импульс, заряд) и неявность функции от координат(это дает некоторый произвол в выборе функций, но так же сильно ограничивает функции определенным классом). Помимо этого симметрии позволяют компактно и красиво записывать уравнения полей и даже строить другие поля исходя из симметрии.
Теперь к элементарным частицам.
Квант это порция, это значим когда мы детектим фотон или электрон, мы получаем не состояния частицы, а всего общего поля(для фотона это будет детектция ЭМ поля, а для электрона электронного поля).
По отдельности мы можем легко составить уравнения поля для ЭМ и электронов, однако если мы захотим состыковать их вместе в единое общее поле, то возникают ряд трудностей. Все дело в симметриях полей. Масса электрона оказывается "лишней" и она вписывается в уравнение движения как совершенно левый/внешний параметр, покушающийся на святое. Для ЭМ поля это не особо критично, все веселье начинается тогда, когда мы начинаем объединять электромагнитное и слабое взаимодействие. Из этого объединения строго следует, что все частицы должны двигаться со световым скоростями, а если мы захотим ввести массу, то лезут чудовищные расходимости.
И тут приходит к Хиггсовскому механизму. Суть в его том, что вводится дополнительное поле, которое нарушает симметрии электрослабого взаимодействия. Ну или другими словами хиггсовское поле взаимодействует с электрослабым полем так, что в уравнение поля возникает дополнительный член, соответствующий массе. Хиггсовское поле наделяет массой только слабое взаимодействие, остальные частицы(электроны/мюоны/кварки/а вот адроны уже отдельная история) уже приобретают свою массу просто из того, что они взаимодействует со слабым полем.
166 512851
Так схерали магнит не влияет на фотоны?

На заряженные частицы влияет.
Но заряженные частицы это те, которые вызывают возмущение в электромагнитном поле, их заряд-это возмущение в электромагнитном поле.
А фотон это возмущение в электронном поле.
167 512852
>>12851

>электронном


*электромагнитном
168 512853
>>12851
Фотон находится на более низком уровне теории нежели электромагнитное поле. Грубо говоря, рассмотрение "фотон в электромагнитном поле", это просто странно. Если ты уж спустился до вторичного квантования, то и определение заряда меняй.
169 512860
>>12853
Я это хотел написать...
Но не дописал. Почему-то не стал.
"И почему у фотона нет заряда?"
15955547308490.png1,8 Мб, 1920x1200
170 512861
А хули хроматографы такие дорогие?
171 512863
>>12861
Евреи...
Лоббируют...
172 512864
>>12863
Пиздос, то бишь никаких редких деталей из чистого золота\платины или алмазных всяких пластин там нет?
173 512868
Поясните пожалуйста за постоянные болезни человека вроде Простуды, ОРВИ и Гриппа. Если например, взять человека и вырастить в пробирке и отправить его заселять другие планеты, при этом звездолет стерилен и с Земли не было занесено никакой заразы, означает ли это что при ослаблении иммунитета из-за холода новый человек на другой планете не будет болеть простудами и ОРВИ? А также всякими условно-патогенными стрептококками и стафилококками? Ведь по сути это инфекция, развившаяся на фоне ослабления иммунитета, но если человек стерилен с рождения, то он не будет этим болеть?
174 512894
Так получается давление вообще никак не приблежает вещество к началу термоядерного синтеза.
Только температура.
Ведь при увеличении давления увеличивается только частота столкновений, за счёт уменьшения объёма.
А скорость и сила столкновений не увеличивается.

А, ну и за счёт частоты увеличивается синтез, это же событие с некой вероятностью, больше частота столкновений-чаще случается синтез.
175 512895
>>12894
Тебе нужно чтобы тепловой баланс соблюдался. Поэтому есть по давлению тоже требования.
176 512898
>>12894
Все проще.
Развивать высокие температуры гораздо легче, чем поддерживать высокое внешнее давление. В добавок при очень высоких температурах, основной вклад в давление осуществляется за счет фотонного газа(давление растет как как четвертая степень).
Вот так и получается, что высокая температура автоматом дает нужное давление.
177 512902
Почему мы не используем вращение нашей планеты как источник энергии?
178 512904
>>12902
Потому что нужна опора, а в космосе вакуум. А на том что вращается толку от опоры нет.
179 512905
>>12902
Затормозим планету и как дальше жить будем?
180 512906
>>12895
Не врубился.

>>12898

>нужное давление


Так что значит нужное давления, оно фактически вообще не нужно и ни на что не влияет, кроме частоты столкновений.
181 512909
>>12906

>давление не влияет


влияет. у тебя слияние происходит при туннелировании черпз барьер. для этого нужно, чтобы частича имела высокую энергию и "ударилась" об барьер. а как ты уже сказал, кол-во ударов зависит от давления. а значит и итоговая "мощность".
на самом деле от давления зависит и среднее расстояние между молекулами. если оно сравнимо с линейными размерами атомов, то оно и через расстояние начинает влиять.
182 512911
>>12909
По сути, давления вообще нет. Это чисто производная наблюдаемая фигня как совокупность процессов которые реально есть.
183 512912
>>12906
Только для идеального газа давление это число ударов о стенку. В общем случае частицы вещества действуют на друг друга дальнодействующими силами и давление нужно переформулировать как плотность работы этих сил. (Ну или поток импульса через ориентируемую поверхность)
Зная уравнения состояния, можно в определенном приближении вывести функцию плотности от давления. Ну или нам даст условное расстояние между частицами вещества, нужное нам для оценки возможности реакции.
Так что высокое давление по-прежнему нужно.

На счет теплового баланса. В расчете реакции нужно знать также термодинамическое поведение всей системы в целом. В реакциях синтеза меняется состав вещества и это выражается в определенных термодинамических эффектах. Может случится, что реакция синтеза окажется невыгодной, хоть по идеи она имеет выигрыш по энергии. Собственно протон-протон реакция очень проблемная в этом плане.
184 512915
>>12911

>давления нет


такими темпами и температуры нет. это производная энергии и энтропии.
185 512916
>>12915
Это было бы забавно, учитывая что без температуры термодинамика, в которой существует энтропия, не может существовать
186 512918
>>12915

>температуры нет


Правильно, есть движение, про которое я и говорил, упоминая температуру.
187 512919
>>12916

>Это было бы забавно, учитывая что без температуры термодинамика, в которой существует энтропия, не может существова



энтропия S = - sum Pi*ln(Pi). обратная температура 1/T =(d/dE) S_max(E). очень странно, как же так. темпа вторична получилась...
188 512920
>>12918
Ну, таки температура - мера теплового движения, что не так? Тип, то, что кинетическая энергия есть мера движения объекта не говорит о том, что кинетической энергии нет
вообще, в термодинамике, в которой движение частиц вообще не рассматривается, температура есть как внутренний параметр, причём такой, что любой другой внутренний параметр можно представить как функцию внешних параметров и температуры
189 512922
>>12919
Ну таки в статфизике первична энтропия, в термодинамике - температура
А забавно это потому, что изначально был термач, и изначально энтропия была введена через температуру (dS=đQ/T)
image.png76 Кб, 1567x375
190 512931
191 512945
>>12902
Почему не используют? Используют, только мало (приливные электростанции). Ебли слишком много ради мизирного выхлопа.
192 512968
Как движущиеся тела(движущиеся каким образом тела) создают гравитационные волны?
193 512977
>>12968
Хуясе, этому целые книги посвящаются, а ты хочешь прям в одном посте ответ?
Упрощенное (сведенное к линейному случаю) уравнение Эйнштейна сводиться к волновому уравнению, это значит существуют решения в виде волн. В общем случае волны уже будут нелинейные и в добавок самодействующие, это одна из причин трудности квантования гравитации. Для создание волны нужно гравируемое тело, движущейся с переменным ускорением: движение по орбите, коллапс или взрыв с большим разлетом массы.
194 512979
>>12977

>Хуясе, этому целые книги посвящаются, а ты хочешь прям в одном посте ответ?


Ну как бы да. По ощущениям это всё формулируется в одном не длинном предложении.
Прочти внимательнее вопрос, может не заметил, я там уточнял.
Не "как они это делают" а "какие тела это делают?"
195 512980
>>12977

>движение по орбите


А вообще возможно в наблюдаемой вселенной движение не по орбите запилить?

Т.е. нельзя же учесть все искривления пространства. Их так дохера, они везде и так сильно перемешаны. от всего.
196 512981
>>12977
И ещё ты напомнил, только не помню точно ли об этом я думал, когда думал "с одной стороны все согласны с Эйнштейном, признают и только опять и опять подтверждают его теорию, но при этом..."

Ищут квантования гравитации?
Разве это не две противоречащих друг-другу вещи?

Или я не об этом думал... Что там ещё у Эйнштейна есть такое, о чём бы я так мог подумать?
197 512982
>>12979

>какие тела это делают?


Почти все, но хоть как либо заметные волны излучают гравитационно связанные массивные компактные объекты, чем ближе к друга друга тем сильнее. Тесные пары белых карликов, нейтроные звезды, черные дыры.
>>12980
Сейчас во Видимой Вселенной нулевой баланс, в том смысле, что в гравитационные силы скомпенсированы другими силами, и "средние" объекты во вселенной просто линейно разлетаются от друг друга.
На масштабах поменьше большая движущийся массы представлена как скучкованный газ, который постепенно охлаждается за счет выбросы своей части в окружающую пустоту.
Только уже на масштабе скоплений галактик есть связанные гравитацией структуры. Так что орбитальное движение локальное явление.
>>12981
Подкину тебе говнеца на пробу: в ОТО не сохраняется энергия.
198 512983
>>12851

>магнит не влияет на фотоны?


Вообще влияет, но не так как на заряженные частицы.
199 512984
>>12905

>и как дальше жить будем?


Поближе к Солнцу.
200 512985
объясните что есть время с точки зрения квантовой физики? Не является ли время еще одной фундаментальной силой? Могут ли существовать частицы - переносчики взаимодействия времени? Потому что для меня даже абстрактные элементарные частицы понятны как сгустки энергии, но что такое время вообще не подается воображению если задуматься. Еще система координат? Но это не отвечает на вопрос. Ведь до Большого Взрыва времени не было и оно появилось вместе с рождением вселенной. к тому же если есть частицы, которые живут миллионные доли секунды, значит есть причина, по которой они живут так мало. Если нашли причину массы частиц, отыскав бозон Хиггса, значит должна быть частица, отвечающая за время жизни других частиц.
201 512986
>>12985

>объясните что есть время с точки зрения квантовой физики?


Время в квантовой теории является динамической переменной. Времени соответствует оператор, собственные числа которого определяют задает величину времени. Этот оператор коммутирует с оператором энергии, и из этого следует что вместе энергия и импульс, связанные собой соотношения неопределенности.

>Не является ли время еще одной фундаментальной силой?


Нет

>Могут ли существовать частицы - переносчики взаимодействия времени?


Бессмысленно.

Короче время это просто расстояние между двумя событиями во Вселенной, не стоит приписывать ему дополнительных магических свойств.

"Большого Взрыва времени не было" - это всего лишь значит мы не можем как-либо измерить расстояния между событиями/состояниями. Если мы будем отматывать назад во времени от текущего момента до Большего взрыва, мы дойдет до нулевого момента, где функция локального расстояния обращаются в ноль - космологическая сингулярность. Через нее нельзя плавно перейти. Так что все ДО для нас физически бессмысленно.
202 512987
>>12986

>Бессмысленно


Что значит бессмысленно? Понято почему стареют и умирают биологические существа. Почему рождаются и умирают звезды. Но атомы то не умирают, они видоизменяются и модифицируются. Вопрос почему субатомные частицы имеют свой цикл жизни? Частица, живущая милионную долю секунды рождается и уходит в небытие, в вечную пустоту не оставляя после себя ничего вообще? Разве не должно быть законов природы, объясняющих почему каждая частица имеет четкий период жизни наряду с массой?
203 512989
>>12987

>Вопрос почему субатомные частицы имеют свой цикл жизни?


Потому что каждая частица - потенциальная яма, из которой энергия может туннелировать в другую яму с какой-то вероятностью
204 512990
>>12982

>Подкину тебе говнеца на пробу: в ОТО не сохраняется энергия.


Это в каком месте?
15965940279990.jpg122 Кб, 639x640
205 512991
Доставьте гайд по изучению математики с нуля
206 512992
>>12991
Есть math/
207 512993
>>12991
Берешь школьнй учебник, например http://verbit.ru/MATH/UCHEBNIK/top-book.pdf
@
читаешь
@
решаешь задачи
208 513000
>>12992
Блять, я так и думал, просто на главной не увидел
209 513004
Реально ли обычному планктону пройти IQ-тест в интернете, получив близкий к истинному результат, или же настоящие тесты есть только у спецслужб?
210 513009
>>13004
Ну, IQ тест показывает, на самом деле, только как хорошо ты проходишь этот самый тест, так что результат всегда близок к истинному
211 513011
>>13004

>к истинному результат


Что в данном случае считать истиной? IQ тест это всего лишь тест дедукции.
212 513031
>>13011
В фильме, который я смотрел на днях, идиократия, показали сценкой типа: Маргарет — "142 IQ", Джефф — "138 IQ". Вот и мне захотелось проверить, дотяну ли я до их показателей (с первого раза, естественно), пройдя тот же самый тест (а не быдляцкую пародию из рунета). Конечно же у актеров, игравших М и Д IQ на десятков пунктов ниже, чем у их персонажей.
Если короче, то я просто хочу знать свой официальный айсикью, который имеют ввиду, говоря об IQ в медиа. Ради интереса.
213 513035
что за знак на втором пике?
image.png61 Кб, 701x213
214 513036
>>13035
бля первый скрин вот
215 513037
>>13036
Существование
216 513040
А каким образом получается, что лупа или вообще любое стекло усиливает свет? Если я построю огромное стекло над солнечной батареей, я получу больше энергии?
217 513043
>>13040
Не усиливает а концентрирует
218 513061
Что будет, если у человека вдруг внезапно исчезнет кишечник? Как его организм начнёт реагировать, выживет ли он? Что придется сделать, чтобы спасти его?
219 513062
Так ведь деревья и трава при гниении выделяет ровно столько же углекислого газа сколько поглотило при росте...
Или не ровно столько же..
220 513063
>>13062
Ну да.
Большие лесные системы и болота обладают почти замкнутым углеродным циклом.
221 513064
>>13061
Кишечник - источник веществ и энергии у организма, помимо этого кишечник выделяет специальные гормоны. Без кишечника организм станет "поедать" сам себя. Конечно, не выживет. Чтоб как-то спасти, нужно давать организму питательные вещества некоторые гормоны прямо в кровь.
222 513065
>>13062
А, вся жизнедеятельность животных это по сути просто часть из вариантов гниянея растений, да, цикл замкнутый.

А то подумао что-то, что типа растяния в ноль выходят, типа выросли и сгнили. А ещё же есть животные, которые подыхают и гниют, но не поглащают со2, в только выделяют.
223 513066
>>13064
А что человек почувствует? Вот, допустим, идёт он по улице, и вдруг у него исчезает кишечник. Почувствует боль? Потеряет сознание?
224 513067
Ждет ли человечество генетический коллапс и тотальное вырождение из-за того что медицина спасает дефективных?
w800.jpg57 Кб, 600x799
225 513069
А Спец. теория отно-ти вообще экспериментально подтверждена, вот чтоб железно? Или это одна из непротиворечащиих наблюдениям математических моделей?
226 513070
>>13069
Эксперимент не может подтвердить теорию. Только опровергнуть (или нет).
227 513071
>>13070
Что ты несёшь? Паули в 1930 выдвинул теорию о нейтрино, в 1956 его обнаружили, т.е. подтвердили.
228 513072
>>13071
оно не доказывает, что теория истинная, а только лишь предсказательную силу. например LIGO с его волнами не доказывает, что ото это истинная теория, т.к не опровергает теорию разумного падения, а просто является еще одним аргументом уровня "мы шатали, а оно не ломается"
229 513076
>>13067
Сомнительно, совсем уж дефектных даже медицина не спасает, и им тёлки не дают, а если и дают, то относительно здоровый всё равно оставит больше детей, так что до полной деградации по здоровью точно не дойдёт.
А вот до умственной-вполне.
Хотя опять же, левочки сдохнут, выродятся в даунов поголовно, а какой-нибудь китай с селекцией без стеснения-просто сотрёт тот биомусор из истории.
Так что в целом с человечеством всё будет хорошо, вопрос кто будет составлять это человечество потом. Явно не все кто есть сейчас.
230 513077
>>13069
А кто решает есть ли подтверждения такими

>вот чтоб железно


или такими

>Или это одна из непротиворечащиих наблюдениям математических моделей?


?
231 513081
>>13072
Т.е. всё, чему учат в школе, вузе, о чём говорят с умным видом умные дяди в умных передачах - просто теории? Та же теория струн, бран и подобного, ну мы ХЗ, может так, а может и не так
15869288624840.jpg39 Кб, 720x576
232 513082
Поясните, что будет, если шахматная программа станет играть против себя? Запустить на 2 компах например, ход с одного делать на другом. Получится ли некая идеальная партия?
233 513084
>>13081
Да, мы ХЗ, есть ли браны, есть ли заряд, есть ли гравитация и тд. У нас просто есть модели, которые хорошо описывают и хорошо предсказывают результат. Например закон кулона. Есть ли поле, или может быть боженька так заряды сам двигает, чтоб мы думали, что есть поле? Наука на это ответ не дает. Наука дает тебе модель с полями и говорит: "оно без ошибок описывает то, что мы видим, если вдруг сломается - пиши. Будем чинить".
234 513085
>>13084

>если вдруг сломается - пиши. Будем чинить


А что чинил Эйнштейн? Разве уже тогда наблюдали что модели что были не работают как надо?
235 513086
>>13085

>а что чинил Эйнштейн


орбиту меркурия чинил.
236 513087
>>13086
Это всё? Или было что-то ещё
237 513088
Чёт как-то не тянет это на переворачивание всей тогдашней физики, только чтобы это объяснить.

Или тогда это была пиздец какая хайповая проблема?
Прям как сейчас..., даже не знаю что.

Что там есть сейчас прям такое необъяснимое, но наблюдаемое?
238 513089
>>13085
Фотоэффекты всякие же, в школе даже проходят.
239 513090
>>13089
Речь о теории относительности.
240 513091
>>13090
Тупо википедия:
Предпосылкой к созданию теории относительности явилось развитие в XIX веке электродинамики[1]. Результатом обобщения и теоретического осмысления экспериментальных фактов и закономерностей в областях электричества и магнетизма стали уравнения Максвелла, описывающие свойства электромагнитного поля и его взаимодействие с зарядами и токами. В электродинамике Максвелла скорость распространения электромагнитных волн в вакууме не зависит от скоростей движения как источника этих волн, так и наблюдателя, и равна скорости света. Таким образом, уравнения Максвелла оказались неинвариантными относительно преобразований Галилея, что противоречило классической механике.

Специальная теория относительности была разработана в начале XX века усилиями Г. А. Лоренца, А. Пуанкаре, А. Эйнштейна и других учёных [2] (см. История теории относительности). Экспериментальной основой для создания СТО послужил опыт Майкельсона. Результаты оказались неожиданными для классической физики того времени: скорость света не зависит от направления (изотропность) и орбитального движения Земли вокруг Солнца. Попытка интерпретировать полученные данные вылилась в пересмотр классических представлений и привела к созданию специальной теории относительности.
241 513092
>>13091
О, спасибо.

Думал там всё вокруг гравитации крутилось при создании, а оказывается вокруг скорости света.
242 513093
Так всё-таки почему раз все согласны с Эйнштейном и его моделью, и опять и опять подтверждают её экспериментально, при этом стараются проквантовать гравитацию и/или пространство, найти квант гравитации и т.д.?

Разве их наличие не противоречит тому как гравитацию объяснил Эйнштейн?

Только для того чтобы опровергнуть теорию относительности найти его пытаются? Или для чего-то ещё?

Они пытаются сделать из квантовой теории теорию всего?
243 513094
>>13092
Гравитация - це ото, цитата про сто (которое це скорость света и релятивистика). Зачем ото, кроме как ради продолжения сто - не знаю, мб на вики тоже есть инфа.
>>13093

>Они пытаются сделать из квантовой теории теорию всего?


Тип того получится, если ото с квантами скрестить.

>опровергнуть теорию относительности


Теории такого плана не опровергаются, а дополняются и переосмысливаются. Будут гравитационные кванты - ото никуда не денется, что ей сейчас можно считать, то можно будет продолжать считать с помощью ото. Но мб починятся баги типа томной энергии, хотя не факт (в т.ч. не факт, что это баг)
244 513095
>>12263 (OP)
Если исчезнет пространство и материя, останется ли время?
Может ли существовать пространство без материи?
245 513096
>>13095

>Может ли существовать пространство без материи?


Да.
Песка, электронов, кварков и собранных из них фигнь нету в одном месте, но прекрасно есть в другом. И может без проблем переместиться туда в первое место где их до этого не было.

>останется ли время?


Не может исчезнуть то чего не было.
246 513097
>>12525

>а наука это попил, развод и наебалово


Не наука, т.к. не использует научный метод.
Ричард Фейнман о научном методе.mp49,7 Мб, mp4,
424x240, 10:05
247 513100
>>13069
Просто посмотри это видео
248 513101
>>13091
молодец, ты спутал ото с сто...
Вопрос дня 249 513106
Товарищи ученые, доценты с кандидатами, дано: грязная одежда, стиральная машина и стиральный порошок.

Допустим, мы хотим очень хорошо постирать одежду и будем для этого использовать стирки два раза – одну программу стирки на полтора часа, вторую на полчаса.

В каком случае одежда лучше отстирается:
– сначала полчаса, потом полтора часа
– сначала полтора потом полчаса
250 513107
Можем ли в теории просчитать движение (спин, массу) всех частиц и на этой основе построить предсказание? Означает ли это, что все предопределено?
251 513108
>>13107
Не можем. Кванты рандомны, неопределенность Гейзенберга не даст тебе

>просчитать движение


детерминизма нет.
252 513110
>>13107
Примерно это делает статистическая физика, но, естественно, не подсчитывает скорость каждой частицы, а изучает закономерности совместного движения всех частиц
253 513111
>>13107
Вопрос уровня /sci/ конечно. Ну ответ такой - Демокрит блеадь, классику знать надо!1!1!! А вот этому >>13108 господину проще представлять мир жиденьким и случайным, чем твердым и однозначным. Из этого своего глубоко упрощенного до абстракции "случайность" представления он делает вывод о невозможности детерминизма, а не о недостатке информации о происходящих процессах. "Если реальность противоречит теории — тем хуже для реальности", тупой верун короче.
254 513112
>>13111
Этот эйнштейн порвался, несите кости
255 513114
>>13112
Введение неопределенности сильно упрощает суть вычислений, обрезая дальнейшее развитие знаний о материи. Если считать поведение саранчи неопределенным, то можно смело ложить хуй на исследование причин перелетов, а изменение окраса и размеров тела списать на магические превращения кварков.
Неопределенность компенсирует дыры в теориях - дыры как бы нет, если поведение объекта неоднозначно. Это позволяет не нести ответственности за откровенный пиздёж.
256 513115
>>13114
Сколько партий с господом богом сыграл?
257 513118
>>13114
Я так понимаю, на то, что принцип неопределённости был введён не чтобы закрыть дыры в теории, а чтобы описать результаты экспериментов, тебе глубоко насрать?
258 513120
>>13118

>Введение неопределенности сильно упрощает суть вычислений


В том числе сами вычисления, забыл написать. Но это очень хуевый способ по изложенным причинам.
259 513121
>>13120

>по изложенным причинам


Каким? "Ета просто", "на самом дили ни так" и "мне нинравится так"?
260 513122
261 513123
262 513124
>>13120
Ты не туда ответил, да насрать
Тот чел неправильно сказал - до квантмеха эту хуйню вообще никто не знал, как считать, чтобы расчёты с экспериментами совпали
263 513125
>>13123
Ну а что там, помимо >>13121?
264 513126
Возможна ли молния в космосе?
265 513127
>>13126
Да.
266 513128
>>13114

> дыры в теориях


> можно смело ложить хуй на исследование причин


Какие дыры и ложить хуй епт? Про это говно как начали сраться в начале века, продолжили во второй половине (неравенста/теорема Белла, который кстати нелюбил Копенгагенскую интерпретацию) так и продолжают сейчас - попытки более точно перепроверить неравенства и опыты по телепортации (квантовой). То что у некоторых людей достаточно крепкие яйца чтобы взглянуть в лицо фактам и признать неопределенность не значит что на проблему "ложат хуй". Ее продолжают долбить.
267 513129
>>13126
да, но это будет нечто своеобразное - света от ионизированного газа практически не будет(по объективеым причинам), а сам разряд не будет нейтральным, поэтому плазмой называться не может, и будет, скорее, электронным газом со всеми вытекающими. возможно даже кванты будут заметнына самом деле они всегда заметны - выход электрона из металла это ~30% за счет туннелирования, но это только при высоком вакууме и низком токе.
268 513130
>>13129

>электронным газом со всеми вытекающими


По напряжению и силе тока будет сопоставимо с настоящей молнией?
269 513131
>>13129
Ток это скорее будет не молния, а что-то вроде катодных лучей в трубке Крукса
270 513132

>>

271 513133
>>12263 (OP)
Продублирую вопрос из /fl.
Жива ли наука на французском языке? Пишутся ли какие-то работы только на французском? В каких областях?
272 513134
>>13133
Щас бы писать не на англюсике и быть живой наукой
unnamed.jpg74 Кб, 900x900
273 513135
Может ли существо иметь очень высокий интеллект, который проявляется в хорошей памяти, быстрой обучаемости готовым навыкам, высоких математических способностях, но при этом быть не в состоянии создать что-то новое путем синтеза имеющейся в голове информации ака изобретать? Есть ли у нас в голове специализированные нейроны, которые позволяют разбирать информацию и соединять с другой подобно лего и в целом быть креативным и находчивым? Я вот знаю, что существовали люди с гипертрофированной памятью или навыками счета, но они при этом были конкретными недееспособными дурачками.
274 513136
>>13135

>Может ли существо иметь очень высокий интеллект, который проявляется в хорошей памяти, быстрой обучаемости готовым навыкам, высоких математических способностях, но при этом быть не в состоянии создать что-то новое путем синтеза имеющейся в голове информации ака изобретать?


Зайди в любой открытый НИИ.
275 513138
>>13133
>>13134
Наврядли, из-за большого количества мигрантов вся западная наука ведется на англ, так проще.
276 513139
Тупой вопрос, а если мы теоретически придумаем как восстанавливать предел Хейфлика, мы получим вечную молодость, но не вечную жизнь? Я о том, что органы (сердце, печень, почки, ЖКТ, мозг...), кости, связки, мышцы все равно будут изнашиваться или нет?
277 513140
>>13139

>кости, связки, мышцы все равно будут изнашиваться или нет?


Я в теме не секу практически, но подозреваю, что уже даже сейчас некоторые компоненты можно заменить на искусственные.
278 513141
>>13139
Не думай об этом, вечная жизнь и так будет у кого надо. А вечная жизнь в этом дьявольском мире была бы сущим адом если ты не животное.
279 513166
>>13134
Ну просто я уже видел книгу по римской истории, которая есть только на французском, но я её случайно нашел, меня другие области интересуют.
280 513171
>>13166
Ну таки зайди, например, в google scholar и поищи на французском
281 513172
>>13171
Понял, спасибо. В общем-то, в этом треде я имею право на тупой вопрос))
image.png7 Кб, 523x184
282 513178
Можно ли, глядя на органическую молекулу, понять, где у неё реакционноспособные центры? Типа такой вижу молекулу органической кислоты, и такой с ходу, О, Кислород, электроотрицателен, нуклеофильное присоединение. О, третичный углерод, значит можно провести реакцию SN1 или как их там блядь. Или смотришь на Пиридин, и видишь, что в орто, пара положения можно нуклеофильные присоединения, а в мета - электрофильные. НЕ ВАЖНО, ЧТО Я НАПИСАЛ В ПРИМЕРАХ.
ВООБЩЕ ПРИНЦИПИАЛЬНО МОЖНО СДЕЛАТЬ ТАКИЕ ПРЕДПОЛОЖЕНИЯ?

Я пытался выучить эту дрисню (органику), но в учебниках тупо перечисление реакций.

Я хочу сделать карточки, где нарисовано соединение, и нужно сказать, по каким положениям оно может реагировать. НЕ ПРОСТО ЗАУЧИВАЯ, а чтобы посмотрел на молекулу, и по основным принципам (индукционный, мезомерный эффект, электроотрицательность, двойные, тройные связи) можно было предположить реакции.

Поэтому ещё раз вопрос:
Можно ли, глядя на органическую молекулу, понять, где у неё реакционноспособные центры?
(и если можно, то может это уже где-то описано?)
283 513179
>>13139
как оно думать, что вечная жизнь клеток не приведёт к вечным мутациям?
284 513181
>>13178
Хз, какие учебники ты читал, обычно они построены на реакциях отдельных групп
Попробуй глянуть клайдена (печатной версии на русском нет, но есть нормальный перевод), там, насколько я помню, неплохо разобраны механизмы
Как говорил один знакомый препод, есть два пути изучения органики - тупо заучивать все реакции подряд, иди изучить все механизмы, но эти оба способа требуют огромного количества времени
285 513184
>>13141
Почему адом?
286 513185
Что будет если поместить проводник в очень сильное электростатическое поле? Вот к примеру если положить в обычное то заряды распределяться по полюсам и внутри напряженности не будет, а что будет если приложить настолько мощное поле что не хватит свободных зарядов проводника его скомпенсировать? Что тогда?
287 513188
>>13082
Нет, они просто настолько вглубь не просчитают. Тем более годные острые игры отсекаються альфа-бетта отсеванием что только делает партию более скучной
288 513195
>>13184
Потому что когда все поймешь, особенно про селедок, впадешь в бесконечную депрессию.
289 513197
>>13195
Не такая уж жизнь и ад, раз ты все понял, а до сих пор не вскрылся.
290 513198
>>13185
Ты только что рентген-трубку
291 513201
>>13185
Начнется эмиссия электронов.
Для этого нужна не так уж большая напряженность - порядка кВ/м в зависимости материала и температуры проводника.
При еще больших начнется испарение самого материла проводника.
292 513202
Аноны, поможите!

The name NMSO4 (en-em-es-oh-four) is a chemical compound, which makes up nitrous nerve gas.

Я совсем не разбираюсь в химии, но как читается это соединение? Понимаю, что сульфат чего-то там.
293 513203
я понимаю, что вопрос тупой, НО.
какую страну анон считает самой свободной в смысле занятия наукой частным лицом или организацией.
просто если открыть УК РФ, то там по некоторым статьям даже от прекурсоров можно уехать на кичу очень надолго.
и все знают как это работает - хранишь что - добрый лейтенант еще добросит, если тебе не хватает до срока из кармана ради палочки и звездочки.
так вот - вопрос касается науки - в какой стране мира ты можешь на своей земле делать что хочешь, если ты не распространяешь там и вреда окружающим не наносишь?
Германия, США, еще что-то?
294 513207
>>13201

>Начнется эмиссия электронов.


>При еще больших начнется испарение самого материла проводника.


Есть ли видео или описание экспериментов с испарением проводника электрических полем?
295 513208
>>13207
С видео не знаю.
Эмиссия электронов - Автоэлектронная эмиссия. Почти в любой книжке про электровакуумной технике найдешь.
А испарение проводника это уже по сути дуга. Вроде используется в ионных напылителях.
296 513209
>>13207
Это, конечно, не научный эксперимент, но тем не менее https://youtu.be/AbpO5cZ1E1Q
297 513213
>>13209
не, тут под вакуумом имеют ввиду низкий вакуум аргона. со всеми вытекающими типа ударной ионизации. по этой херне 1001 книга есть. а вот разряд в высоком вакууме, где есть только электроны и ионы электродов... я про это даже в сивухине не видел...
298 513214
>>13209
Был подписан на этого додика, когда у него было меньше 1000 подписчиков. Тогда он снимал охуенные видосики по пиротехнике и морду свою не палил. Отписался, когда стал популярным, начал снимать хуиту и рекламировать танчики/самолётики/викинги.
299 513226
У меня охуенная идея. А если делать провод, где вместо медного проводника использовать плазму? Как это можно использовать?
300 513227
>>13226

> у меня охуенная идея


> как это может использоваться?

301 513230
>>13226
ты только что придумал разрядник. использовать можно как разрядник. на самом деле еще как лампу и волновод. возможно изза высокой длины там может появиться пара интересных явлений, поэтому её можно в нии построить. зачем? другой вопрос.
302 513233
Как получается, что красный и синий дают фиолетовый? Идентичен ли он фиолетовому в радуге, или мы просто так его воспринимаем
303 513235
>>13233
Мы его так воспринимаем. Не идентичен.
304 513252
Помоги с проблемой, сайфач. Я в рамках дипломной работы магистра сделал дозиметр. Мне нужно найти какой-либо фонящий объект для его исследования, пока что нашел только гранитный памятник на кладбище с фоном 80-100 мкР. Слышал про калильную сетку от газовой лампы с оксидом тория, но нашел только китайский хлам из асбеста. Где я могу еще найти вещи/вещества которые были бы не оч дорогими, но фонили?
305 513253
>>13252
Чекни канал Радиоактивные материалы. Там чел на алике пару фонящих вещей находил (и не только на алике).
306 513268
>>13253
От души, братик. Большое тебе спасибо.
307 513269
>>13252
Торговые электроды
308 513270
>>13269
*ториевые, сорян
309 513271
Почему космическая пыль прозрачна для рентгена?
310 513272
Сап двач. По закону Бернулли чем быстрее течёт вода тем меньше давления он создаёт. Тогда почему автомойки убирают любую грязь? Или он получается работает только если вода в трубе, а на то что снаружи не распространяется?
311 513273
>>13272
Во-первых закон актуален для гладких потоков.
Во-вторых давление определяет всего лишь какую работу совершит ток жидкости на сам себя или границу. Этакая плотность потенциальной энергии потока. Ключевое слово поток.
В случае автомоек поток негладкий и очищает от грязи за счет механического удара частиц жидкости/смеси. Силу удара определятся только скоростью и плотностью частиц. Если мы будем действовать на грязь давлением, то вместо удара грязь тупо войдет в равновесие с потоком.
312 513274
>>13272

> The principle is only applicable for isentropic flows: when the effects of irreversible processes (like turbulence) and non-adiabatic processes (e.g. heat radiation) are small and can be neglected.


Струи воды летящие по воздуху само собой являются турбулентными.
313 513275
Я слишком запутался в терминологии, поясните

1) Как связано существование NP-полных задач и существование односторонних функций? Вообще односторонние функции могут не существовать, если P не равно NP?

2) Какими будут последствия, если класс NP содержит ровно два класса P и NP-Complete и не содержит иных классов? В частности, последствия для криптографии?
314 513276
>>13203

>в какой стране мира ты можешь на своей земле делать что хочешь, если ты не распространяешь там и вреда окружающим не наносишь?


Мексика и все что южнее. У местной полиции реальных проблем хватает, им не до тебя будет.
315 513277
>>13275
1)однострониие функции это функции, которые NP, но не Р.
2) это значит, что все неполиномиальные задачи можно "перевести" одну в другую за полином. при этом ни одну такую задачу нельзя эффективно решить на классическом компе.
316 513306
Скажите мне, пожалуйста, как правильно назвать некий абстрактный алгебраический конструкт, который объединяет в себе группу множеств с общими свойствами.
317 513307
>>13203

>Германия, США


Ты назвал два police state, поздравляю.
318 513308
>>13306
Теория категорий.
319 513312
>>13308

>раздел математики, изучающий свойства отношений между математическими объектами


Не. Это раздел. А мне нужно множество над множествами или алгебра над множествами, или над какими-нибудь кольцами, я хз.
320 513314
>>13306
Внезапно множество множеств https://proofwiki.org/wiki/Definition:Set_of_Sets
321 513351
>>13314
>>13312

множества всех множеств не существует. это либо шайтанский юнивёрсум, либо категория set. второе удобнее.
322 513357
>>13351
А ему всех и не надо, он же сказал "с общими свойствами".
323 513358
>>13357
тогда это просто какая-то категория, а не set
324 513419
>>13270
спасибо, бро с:
325 513434
Почему у меня под психоделиками (небольшими дозами) дико обостряется восприятие и понимание сложных концепций, сюжетов, музыки, текстов. Улучшается восприятие цвета и звука. Я могу чувствовать себя как ребенок, возвращается то детское состояние когда весь мир был интересен, забываются проблемы и тревоги, каждая вещь интересна, хочется все изучать, читать, смотреть.
Без психоделиков я тревожный, нечего не интересно и нет желания что либо изучать (я изучаю, развиваюсь но вот этого детского любобытства и интереса нет).
Почему в обычном состоянии все так хуево и уныло.
326 513438
>>13434

>возвращается то детское состояние


Зис. Ты ж сам себе ответил.
Возвращается не разъёбанный ещё детский гормональный баланс.

>Без психоделиков я тревожный, нечего не интересно и нет желания что либо изучать


А это нормально, ты же прошёл через отечественную школу и вузик, они свою работу сделали.
327 513441
>>13434

>дико обостряется восприятие и понимание сложных концепций, сюжетов, музыки, текстов.


1) Какие это нахуй *сложные концепции"?
2) На основании пункта 1, ты не начинаешь что-то там понимать, потому что это хуета без какого-то сложного смысла, ты просто начинаешь маняфантазировать, и не в том даже смысле что выдумываешь там какие-то смыслы, а начинаешь фантазировать о том что якобы выдумываешь/выдумал, хотя на самом деле нет.
328 513445
>>13272
А вот кстати, закон Бернулли это не пиздёж?
Вычитали как-то влияние высасывания давления из монометра потоком жидкости?
В быстром потоке реально давление меньше, чем в части этого же потока в широком месте трубы?
329 513447
>>13272
Кинетической энергией очищает. Быстролетящая куча воды ударит по грязи сильнее чем медленно летящая.
е=mv^2/2
330 513448
>>13271
Не прозрачна.
331 513449
>>13227

>и как это сделать



У меня охуенная идея-телепортация. Мнгновенно и в любую точку вселенной. Как это можно использовать? И как это сделать?
332 513450
>>13179
У мышей рак также редко встречается как и у людей до 2 лет?
333 513455
>>13448
Тогда почему центр галактики наблюдают в рентгене?
334 513457
>>13445
Закон Бернулли верен для ламинарного потока изотропной жидкости/газа. С вихрями или неоднородными среда уже другая история.
Закон выводится из законов сохранения и простой модели непрерывной среды, хотя изначально закон был эмпирическим.
Да, и на законе Бернулли основано много технических устройств. К примеру, крыло самолета.
335 513496
>>13457
про крыло самолета - заблуждение. там гораздо более сложные законы со всякими циркуляциями скорости потока и прочими струйными колдунствами.
336 513513
Почему благодатный огонь сначала холодный, а потом обжигает? Как он самовоспламеняется?
337 513514
>>13513
потому, что нет нормальных пруфов и поэтому сведетелей нельзя уличить во лжи.

по божественной воле. или не сам.
338 513515
>>13277
1) То есть проблема существования односторонних функций и проблема равенства классов P, NP эквивалентны между собой (из существования односторонних функций следует P != NP, из P != NP следует существование односторонних функций)?
Я думал, что проблема односторонних функций сильнее, и даже если их не существует, скорее всего P != NP

2) А какими будут последствия для криптографии? Ведь криптография на NP-полных задачах, как я понимаю, не популярна, то есть практически не существует, потому что для NP-полных задач существуют практически приемлемые приближенные алгоритмы, которые иногда, пусть и очень редко, дают ответ, совпадающий с точным. (Поэтому пусть и редко приватный ключ удастся извлечь из публичного. Для разложения чисел, например, такого варианта есть, там либо есть ответ, либо нет)
339 513517
>>13496
Не такое уж заблуждение. Формула Жуковского выводится из закона Бернулли переформулировкой потока и давления при помощи теоремы Стокса.
Подъемная сила обеспечивается разностью давлений снизу и сверху, возникающая за счет разности скорости потока.
Другое дело как крыло ускоряет поток сверху и замедляет снизу.
340 513524
>>12263 (OP)
внедрение ии и нейронки осталым смогут ли они в обучение? и псих неполноценным людям смогут ли они вернутся в общество как этот будет проихсходить , будет ли вообще и как?
341 513526
Кто шарит за ядерную физику, скажите он поехавший или нет.
https://www.youtube.com/watch?v=7SHE4v_DEGs
342 513527
>>13526
Если верить вики, то создается впечатление, что с дедком происходят очень странные веществаhttps://ru.wikipedia.org/wiki/Острецов,_Игорь_Николаевич
343 513529
>>13526
Вот с боярышником интересное видео
https://www.youtube.com/watch?v=WMpAST_S7eE
344 513533
ищу книгу - Кристофер Поттер: Вы находитесь здесь. Карманная история вселенной
345 513534
>>13529
думал ахуенный дед , а оказалось не совсем, воду льёт и рекламу ебашит как не в себя.

Так-же был замечен за лицемерем, есть у него на стримах пизда какая-то, я нюфаг, не знаю кто, мож дочка. Кароче, начало стрима вырезать забыли, и она на него бочку катит, хули ты старый сиди тут с таким ебалом, с таким ебалом сидеть не надо, веди себя весело, такое тут не нужно, и как ебанутая начинает классическую "ПРИВЕТ ДОРОГИЕ ДРУЗЬЯ" и дед типа, да да, привет, друзья, спалились конечно они ахуительна.
346 513536
>>13534

>спалились конечно они ахуительна


Ты недавно на ютубе? Там только ленивый не прохаживался на тему того, как заёбывает натужно улыбаться на камеру, а без натужной улыбки никто не смотрит, и сколько ютуберов коксом заправляется перед записью.
347 513537
>>13434
Мне кажется ты путаешь психоделики и психостимуляторы/психоактивные вещества в принципе. Но вообще говоря это, и если не вдаваться в подробности (они и невозможны без точного понимания вещества) - строго говоря это иллюзия. Как во сне тебе может казаться что говорящая свинья, ходящая у тебя по дому например, это абсолютно нормально - так и тут.

>Почему в обычном состоянии все так хуево и уныло.



Потому что у тебя развилась психологическая зависимость.
348 513538
>>12266
очень интересно почему?
349 513573
>>13526
Дайте выдержки хоть, а то смотреть зогошную муть сил нет.
350 513578

> это парадокс, но никак не для того, кто знает физику, так как здесь работает один из ее законов.



Это же жирнота?

https://zen.yandex.ru/media/id/5dc70704902e62533e35ace5/kak-za-5-sekund-ohladit-arbuz--laifhak-5ee68ab47c13c822dba28b30?from=feed&utm_referrer=https://zen.yandex.com&rid=2693493799.553.1597613419954.56987&integration=yandex_launcher&place=more&secdata=CO2xzqSrLiABMAJQyAE=
351 513581
>>13578

>Это же жирнота?


Скорее глупость, автор искренне верит в свой бред. Горячее тело (кипяток) не может забрать тепло у относительно холодного теда (арбуз), поэтому кипяток его только нагреет. Однако кожура у арбуза весьма толстая поэтому за 5 секунд внутренности прогреться не успеют. И тут уже может получиться сенсорная иллюзия - прикоснувшись рукой к горячей корке, а затем к холодной мякоти, мякоть покажется холоднее чем она есть на самом деле, но абсолютная температура мякоти по факту такая же какой была бы и без кипятка.
352 513582
>>13536
при чем тут ютуб, деда дрочит какая-то тня, вот в чем, если бы дед сам это делал, то окей, какие вопросы
15939212825570.jpg26 Кб, 680x289
353 513584
С чего начать изучение такой науки как физика? Хочу стать инженером.

Сам пока что изучаю математику до calculus, потому что в американской системе рекомендуется переходить к физике после основательного изучения математики. А отечественная школа меня ничему не научила.

Проблема в том, что в математики меня привлекает только навык быстрого и точного счета. Как я понял, что в физике всё расчеты всё равно будут проводиться с помощью калькуляторов, не так ли?
354 513586
>>13584
Восполняй пробелы в математике по необходимости. На самом деле математики там не так уж и много, но хотя бы с этим немногим надо уметь уверенно работать. Никто (кроме ебнутых фантазеров-псевдофизиков с их мультивселенными и 20-ю измерениями) не требует от тебя знать больше чем самые основы: векторы, матрицы, аффинные преобразования, производные, интегралы, дифуры, тригонометрия, комплексные числа и т.д. При этом точные определения не нужны, просто умей всем этим пользоваться.
355 513589
>>12465
Ого, ты открыл второй закон термодинамики
356 513590
>>13586
А что делать с тем, чтобы понимать физические законы и принципы?

Часто на улице думаю в голове - было бы круто, если бы я точно знал формулы и причины движения автомобиля... Или почему я слышу пение птиц так плохо(для меня птицы поют какими-то отрывками, будто каждый звук это либо 1 либо 0. Полутонов не слышу).

Я не знаю как объяснить точнее, но мне хотелось бы, чтобы моя картина мира была адекватной и проверяемой, а также - прогнозируемой.
357 513592
>>13590
Просто представляй всё как набор элементарных частиц.
Знать всё не нужно, и почти все формулы эмпирические, а не фундаментальные, о которых ты думаешь, это не какие не "законы".
358 513594
>>13592
Со школьной скамьи я помню, что есть "математические законы физики", а есть разные "константы", которые могут поменяться, если меняется окружение.

Что значит "представляй себе набор элементарных частиц"?
359 513596
>>13594
Почитай многотомник Сивухина.
360 513597
>>13584
На твоем месте, я бы не слушал таких как >>13586, так как по его мнению, любой 11 класник является физиком. Это естественно не так.

>Хочу стать инженером.


Инженеры бывают пиздец какие разные. Поэтому тут нужны пояснения, каким инженером. А так набор стандартный: какой-нибудь Stewart Calculus, что-нибудь по линейной алгебре, а дальше выход в ОДУ(ODE) и плавный переход к ДУЧП(PDE) и вычислительным методам(метод конечных элементов и прочее) Также придется освоиться с каким-нибудь COMSOL. Но это самая основа-основ, возможно придется заглянуть также в теорию групп, в представления групп, функан. Дальнейшеее зависит от области специализации.
361 513600
>>13596

>Почитай многотомник Сивухина.


А не устарел?
>>13597
Инженер-робототехник, если быть точнее. Понимаю, что тут надо мехатронику, электронику и программирование, в основном. Может ещё в оптику.

Уже прохожу курс по работе с Ардуино(база про работу с электроникой, курс нравится, потом думаю купить и настоящие планки, а не только в эмуляторе играться).

>COMSOL


Загуглил, пока не нашел для него бесплатных курсов. Как я понял - это программа в которой объекты подвергаются физическим тестам? А сам объект создавать в Autodesk Fusion 360 ?

Насчет перечисленного матана. Как я понял - всё входит в Calculus Ab + Calculus BC ? Матан прохожу вот тут https://www.khanacademy.org/math
362 513601
>>13600

>А не устарел?


Как общая физика может устареть?

> Как я понял - это программа в которой объекты подвергаются физическим тестам?


Если кратко, то решение ДУЧП на сложных геометриях. Т.е физическая симуляция простейших(и не очень) процессов.

>Как я понял - всё входит в Calculus Ab + Calculus BC ?


Конечно нет. Линейная алгебра это отдельный предмет. ОДУ иногда запихивают в калькулюс, однако это тоже несовсем, как по мне верно. Численные методы тоже являются отдельным предметом. По всему этому придется читать тонну страниц.
363 513602
>>13534
а можна ссылку на этот видос?
364 513609
>>13602
да у него на канале был, я уже хуй вспомню что это было, скипнул деда давно
365 513668
Диаметр вселенной в световых годах значительно больше ее возраста, значит она расширяется или расширялась со сверх световой скоростью. Почему это не привело к временному парадоксу (см. тахионный телефон)?
366 513670
Почему человек рождается зрячим, когда щенки и котята слепыми.
Человеку в младенчестве ничего не грозит, он еще долго лежит на спине и ждет когда ему в рот ткнут сиську. А новорожденные животные должны карабкаться к соску расталкивая братьев, должны уползать в безопасное место если опасность.
367 513671
>>13670
Тому что у человека куда менее развито обоняние по сравнению с собачьими и кошачьими.
368 513672
>>13671
Это не ответ
369 513673
>>13672
Только если ты дебил.
370 513674
>>13673
Недебил, распиши как это

> Тому что у человека куда менее развито обоняние по сравнению с собачьими и кошачьими.


Связано с этим

> Почему человек рождается зрячим, когда щенки и котята слепыми.


Или у тебя понимание мира на уровне сообщающихся сосудов
371 513675
>>13674
Для таких, как ты, у нас есть Б-г. Люди рождаются зрячими, потому что Б-гу угодно было так сделать. Пшол нахуй.
372 513676
Как определили строение атома?
Правда, что элементарных частиц больше трех?
373 513677
>>13675
Да бля, а Гребенщиков-то тут при чем?
374 513678
>>13675
Тебе место в майл ответах а не тут
375 513679
>>13668

> тахионный телефон


В телефоне проблема что сигнал приближается к цели со сверхсветовой (если взять систему остчета связаную с сигналом то цель приближается со сверхсветовой), а при расширении вселенной цель всегда удаляется (в системе отстчета связаной с сигналом).
376 513681
>>13272

>По закону Бернулли чем быстрее течёт вода тем меньше давления он создаёт. Тогда почему автомойки убирают любую грязь?


Потомоу что при соприкосновении с авто скорость потока падает, отчего давление резко возрастает.
377 513692
Ну так что там, условиях ковидла снизились темпы глобального потепления?
378 513696
>>13692
А то! От того, что все сидят дома за 300-ваттными пекарнями и 2к-ваттными кондиционерами вместо сидения на работе за 50-ваттным тонким клиентом и одним промышленным кондеем на 50 человек, глобальное потепление умерло от ржаки.
379 513697
>>12263 (OP)
почему нет ни разговоров ни проектов космических кораблей основанных на силе ампера?

в дополнение: разве плазма и другие сверхкритические состояния не позволяет получить мульёны ампер в каком то теле жидкие металлы вот это вот всё
380 513699
>>13696
Аааа, раньше промышленность, потом коровы, теперь компьютеры виноваты?
381 513702
>>13699
Так это ж считай обогреватель. Получаем 300 ватт из розетки - 300 ватт уходит в нагрев радиаторов. Даже летом. Это же пиздос.
382 513704
>>13697
Нет. Не позволяют
383 513708
>>13697

>почему нет ни разговоров ни проектов космических кораблей основанных на силе ампера?


Ионный двигатель же. Правда там вроде как электрическое отталкивание, но может где и магнитное есть, суть та же.
384 513710
>>12263 (OP)
Существует ли абсолютный электрический потенциал?
Возьмем кусок металла в вакууме, будем сообщеть ему электрический заряд. В какой-то момент его молекулы начнут ионизироваться, и заряд перестанет быть только поверзностным. Можно ли так определить его заряд не как разность потенциалов, тоесть без второго тела?
И если возможно, можно ли так определить разность потенциалов между несвязанными друг с другом телами, поднося к каждому по такому вот эталонному куску металла?
385 513715
Постоянно ли ускорение расширение пространства-времени?

Массивные тела подтаскивают к себе обратно соседей обычной гравитацией, которые норовят свалить в результате расширения?

На каких дистанциях эффект расширения уже ощутим?

Например в масштабе галактики, представим звезду которая наяривает по внешнему краю.
Получается ли что 4-я космическая будет чуть-чуть меньше чем расчётная за счёт того её помогает свалить в том числе расширение пространства времени?

Или в масштабах галактики этот эффект всё ещё незаметен и аткуально оно только на межгалактических масштабах?

Поэтому я спрашиваю постоянно ли ускорение расширения
- мне интересно в далёком будущем будет ли вклад от эффекта расширения всё более и более значимым, что бы даже начать оттаскивать планеты от звёзд?

И ещё вопрос - пространство-время сжимается под действием массы.

Можно ли как-то увязать глобальную степень сжатия пространства-времени с массой вселенной?
Математичсеки не выглядит ли это так, будто суммарная масса вселенной по какой-то причине снижается?
386 513719
>>13715

>Постоянно ли ускорение расширение пространства-времени?


Неизвестно, но наиболее унылый сценарий, что да.

>Массивные тела подтаскивают к себе обратно соседей обычной гравитацией, которые норовят свалить в результате расширения?


Да, главное чтоб за определенную границу не убежали.

>На каких дистанциях эффект расширения уже ощутим?


Уже на масштабах скопления галактик, впрочем он измерим и на более мелких, если бы не темная материя.

>Получается ли что 4-я космическая будет чуть-чуть меньше чем расчётная за счёт того её помогает свалить в том числе расширение пространства времени?


Да, но на масштабе галактики слишком малый эффект.

>мне интересно в далёком будущем будет ли вклад от эффекта расширения всё более и более значимым, что бы даже начать оттаскивать планеты от звёзд?


Неизвестно, но скорее всего темной энергии силенок хватит только на разрушение скопления галактик.

>пространство-время сжимается под действием массы.


Оно не сжимается. Деформация 4-объема геометрически довольно сложно описывается, нежели сжатие/сдвиги. Попутно принцип эквивалентности накладывает свои ограничение на эти деформации(локальное сохранение 4-объема). И в ОТО искривляет не масса, а поток импульса-энергии.

>Можно ли как-то увязать глобальную степень сжатия пространства-времени с массой вселенной?


Нет, мы можем только определить среднею плотность.

>Математичсеки не выглядит ли это так, будто суммарная масса вселенной по какой-то причине снижается?


Нет не выглядит, потому что при попытках считать массу надо надо задать замкнутую гиперповерхность. В расширяющийся вселенной гиперповерхность может спокойной уйти на бесконечность, но суммарная масса сохранится.
387 513721
Как определили из чего состоит атом?
Как определили альфа, бета и гамма-излучения?
Есть ли частицы, которые меньше протона, нейтрона и электрона?
388 513722
>>13721

>Как определили из чего состоит атом?


Коллайдер + детектор. И определили это сильно сказано. Скорее предположили в рамках текущей модели.

>Как определили альфа, бета и гамма-излучения?


Это три совершенно разные вещи. Ядро гелия, электроны/позитроны и фотоны. Различаются массой, импульсом, скоростью, спином, энергией.

>Есть ли частицы, которые меньше протона, нейтрона и электрона?


Разумеется. Фотон ты уже знаешь например, у него вообще массы нет. Кварки-лептоны гугли сам. Можешь ещё что-нибудь про детекторы поискать https://www.desy.de/~garutti/LECTURES/ParticleDetectorSS12/L1_Introduction_HEPdetectors.pdf
389 513723
>>13721
Ты типа открыл учебник по физике за девятый класс, увидел что там есть это, и решил вместо того, чтобы просто прочитать, попросить на дваче тебе скопировать текст из учебника? Зачем?
390 513724
>>13722
>>13723
Я открыл сайт "физичка". Там было написано, что просто взяли уран и рядом магнит. А потом смотрели, где был засвечена пленка. Или не уран.

Короче, я потому и спросил, что там было написано слишком не подробно, а в википедии слишком сложно.

>Скорее предположили в рамках текущей модели.


А что современная модель говорит про связь волны и частицы? Я вообще не понял этот момент. Свет волна? Свет частица?
А другие элементы и тела это тоже волна и частица? Например как работает запах? Мы чувствуем молекулу? То есть уловил себе какое-то количество молекул и чувствуешь запах? Может ли запах быть тоже связкой частицы и волны?
391 513725
>>13724
Частица и волна это старые модели. Новой похуй, она там и то и то одновременно.

>Например как работает запах? Мы чувствуем молекулу? То есть уловил себе какое-то количество молекул и чувствуешь запах? Может ли запах быть тоже связкой частицы и волны?


Химическая реакция. Молекула просто дергает твой рецептор.
392 513726
>>13725

>она там и то и то одновременно.


Как она может быть одновременно и тем и тем?
Что такое рецептор в твоем понимание?
393 513728
Допустим время работы двух каких-то людей стоит одинаково, так совпали их навыки, умения, знания, цены на рынке, издержки работы и т.д.
Без налогов ты просто меняешь 1 час своего рабочего времени на 1 час рабочего времени другого человека.
А если налоги 30% с дохода?
Получается при налогах 30% ты сможешь поменять 1 час своего рабочего времени на 1x0.7=0.7 0.7x0.7=0.49 часа работы другого человека?
Или только один раз надо на 0.7 умножать, и ты сможешь поменять 1 час своего времени на 0.7 часа рабочего времени другого?
Или и не 0.7 а ещё какое-то другое будет?
394 513729
>>13728
ты сначала говоришь про двух каких-то людей которым работодатели платят одинаково платя одинаковые налоги.

А потом спрашиваешь про себя самого, который работает на себя и имеет откуда-то прибыль и платя уже какие-то иные налоги или не платя их вовсе и думаешь про наём?

Определись с начальными условиями.
Учти что если один человек работает на дядю а второй "на себя", то у второго кроме "рабочих" скилов есть ещё "предпринимательские" скилы / связи / итп - т.е. он уже более дорогой чем просто наёмник.
395 513730
>>13726

>Как она может быть одновременно и тем и тем?


Я бы вообще сказал, что не то и не другое. Короч, наши понимания частицы и волны слишком простые для сложного реального микромира. Мы придумали модель "частицы" и вроде как ее поняли, аналогично с волной, а на деле есть что-то еще, которое нашей интуиции не поддаётся. Мы можем подгонять в некоторых моментах это "что-то" под свойства частиц или волн и говорить, что это "что-то" и частица и волна; а на деле это просто другая хуйня.
396 513731
>>13729
Блять, вот нахуя эту пустую демагогуэию разводить? Что там непонятно?
Лишь бы попиздеть.
397 513732
От чего зависит скорость волны?
В воде
398 513734
Чёрные дыры могут создавать массу не из чего?

Допустим кидаем камешек с метра на нейтронную звезду, он разгоняется до 100 тыс. км в секунду перед ударом.
Выделится энергия большая.
При этом масса не потратится, энергия гравитационная перейдёт в излучение.
А если это будет чд.. а, ну тогда удара не будет, и выделения энергии, ладно.
Хотя можно придумать что-то типа столкновения с другим предметом ещё до горизонта событий, но уже под сферой, из которой фотоны могут убежать.

Но тогда возвращаясь к нейтронным звёздам и киданием камешка на них, как-то непонятно откуда берётся энергия. Камешек и раньше был отдалён от нейтронной звезды, его с неё не подняли, и если бы вместо нейтронной звезды тут была обычная, или вообще газо-пылевое облако, т.е. камешек бросили просто чуть раньше, никакой такой большой энергии не выделилось бы.
399 513735
>>13734

>Допустим кидаем камешек с метра на нейтронную звезду


Дальше не читал.
400 513736
>>13735
Ты чего порвался?
401 513737
>>13736
Человек говорит о гипотетических объектах на полном серьезе, плюс не понимает что такое гравитация.
402 513738
>>13737
Ты похоже пересмотрел научпоп-видосиков.
403 513739
Может ли волна "свернуться в частицу"? Или частица разобрана на волну?
Струнная теория имеет какое-то применение?
404 513740
>>13726

>Как она может быть одновременно и тем и тем?


Ну вот так. Там сложные математические конструкции. Их нарисовать не получается, и на пальцах объяснить тоже, ибо там бесконечномерные пространства.
405 513741
>>13739
Нет. Нет.
Да.
406 513742
>>13740

>ибо там бесконечномерные пространства.


Что это за дичь?
407 513743
>>13742
Обычное линейное пространство, только с бесконечной размерностью.
Самый простой пример это пространство всех функций. Из матанализа любую гладкую функцию можно представить степенный рядом.
408 513744
>>13743
А теперь объясни для тех, чье развитие в математике закончилось на алгебре.
409 513745
>>13744
Да блядь. Тут целый первый курс технического вуза.
410 513746
>>13745
В каком вузе меня этому обучат? Мифи? Мгу? Бауманка? Физтех? Политех?
411 513747
>>13746
Да хоть в усть пиздейском.
Матан и линал минимум для любой технической специальности, даже в ПТУ.
412 513748
>>13747
Как я понял, ты изучал матан и линал. Значит можешь рассказать про эти многомерные пространства.
413 513749
>>13744
Если совсем просто, то смари
У тебя есть зависимость функции от одного параметра f(x)=x
Для того, чтобы нарисовать график функции тебе нужно двумерное пространство (график, тогда будет y=x)
Если f(x)=x+y
То тебе надо уже трёхмерное пространство
И так далее, если три параметра, для отображения графика тебе понадобится четырёхмерное пространство
Если разложить функцию в ряд, её можно представить как бесконечный ряд, и получается, что функция зависит от бесконечного количества параметров, а значит и пространство бесконечномерное
математики, сильно не бейте за такое пояснение
414 513750
>>13748
Ту дело в том, что их изучают весь первый курс
Ты хочешь, чтобы тебе на дваче первый курс вуза расписали?
415 513751
>>13749

>значит и пространство бесконечномерное


А что это дает в практическом смысле?
416 513752
>>13751
Разложение в ряд - возможность представить сложную функцию как сумму простых, и вычислить приблизительное значение (там чем больше члены ряда возьмёшь, тем точнее будет значение)
Бесконечномерные пространства - это просто объект в матеше, может они и используются в каких-то областях, но вот я в душе не ебу
417 513753
>>13752
Вроде бы понял, спасибо.
418 513754
>>13731
Задачу хуёво поставил, исходные данные противоречивые и неполные.
30% налог с какого блядь дохода, с твоего как нанимателя?
С зарплаты твоего наёмника (его зарплата для тебя нихуя не доход а расход)?
Если ты наниматель то с какого хуя тебе кто-то платит столько же как второму хую которым ты себя хочешь подменить? Это типа на фрилансе Б2Б и заебался сам работать, хочешь типа аутсорсинговую компанию сделать? Учти что если ты на ИП - там лимиты по найму, выше которых нужно уже норм юрлицо.
Пиздуй короче в /biz
419 513757
Существуют ли какие-то теоретические пределы длины ЭМ волны?
Самая длинная из возможных?
Самая короткая из возможных?
Если такие пределы есть - что мешает иметь больше/меньше?
Если пределов нет
- что из себя будет представлять ЭМ волна с бесконечно длинной?
- что из себя будет представлять ЭМ волна с нулевой длинной?
- можно ли говорить о фотонах в таких вот граничных случаях?
420 513761
>>13757

>Существуют ли какие-то теоретические пределы длины ЭМ волны?


Нет. Но поскольку во Вселенной существуют иные объекты помимо ЭМ волн, да и Вселенная очень даже конечна, чтоб вместить нужную волну.

>Самая длинная из возможных?


Во всю наблюдаемую вселенную, а так ограничивается только точками, которыми можно соединить светоподобных интервалом.

>Самая короткая из возможных?


Планковская, волны с такими длинами уже начинают гравитационно взаимодействовать с друг другом. Впрочем из-за того, что в ЭМ поля являются частным случаем электрослабого поля, кванты с очень которыми длин волн перестают вести как чисто электромагнитные. В добавок есть предел связанным с тем, что не найдется процессов, способные излучить такие короткие волны.

>что из себя будет представлять ЭМ волна с бесконечно длинной?


Просто статический потенциал.

>что из себя будет представлять ЭМ волна с нулевой длинной?


Функция Хевисайда.

>можно ли говорить о фотонах в таких вот граничных случаях?


Квантование как раз придумали, чтоб эти мутанты не возникали.
421 513762
>>13761
Круто, спасибо.
422 513764
Куда поступать, если я хочу стать Теслой XXI века?
423 513765
>>13764
во взрослую жизнь вступи сначала, а то больно будет с такими маняфантазиями
424 513779
Скажите, есть ли смысл вкатываться в нейронауку с целью добиться своего бессмертия в будущем?
У меня сейчас такое чувство, что загрузку сознания в том или ином виде изобретут в ближайшие 20-30 лет, но из-за чудовищного числа желающих и сложности процесса доступна она будет лишь 0.1% населения Земли.
Так вот я хочу подготовиться к тому, чтобы, если так случится, понять общий принцип и воспроизвести процесс самостоятельно.
425 513782
Возможны ли ирл экуменополисы типа корусанта из зв? Ну или хотя бы просто огромные многоуровневые города? Они не обвалятся под своим весом? Если нет, то возможна ли будет жизнь на самом первом уровне? Там вообще кислород будет, если уровней, допустим 200-500?
426 513783
>>13782

>Они не обвалятся под своим весом?


Смотря какая архитектура, например если это обычный город но каждый дом в нем - небоскреб.

>Там вообще кислород будет


При искусственной поддержке кислород может быть где угодно, как и прочие блага. Спустись ав шахту - там вполне себе можно дышать.
427 513784
>>13710
Бамп вопросу.
428 513787
>>13779

>понять общий принцип


На ютубе расскажут, не ссы.
429 513788
>>13710

>Существует ли абсолютный электрический потенциал?


Да, и он равен нулю.
430 513791
>>13788
Я не о суме потенцилов в замкнутой цепи говорил, прочитай пример.
431 513793
>>13779
Толстовато.

>У меня сейчас такое чувство


>в ближайшие 20-30 лет


А ты не думал, что у живших до тебя людей тоже были такие чувства? Нет, возможно и изобретут, пути господни неисповедимы, лол. Но ты скорее всего просто осознал свою смертность и испугался, что после этого никакого "тебя" не будет. Это бывает.

>понять общий принцип и воспроизвести процесс самостоятельно


Ну общий принцип, скажем, светодиодной лампы может понять и ребёнок. А воспроизвести её?
432 513796
>>13791
Относительность эм потенциала одно из требований калиброванных теорий. Тут история примерно как с абсолютным пространством Ньютона. Может и можно как то измерить абсолютный потенциал, только вот смысла от этого не будет.
433 513797
>>13779

>загрузку сознания в том или ином виде изобретут в ближайшие 20-30 лет


Не изобретут никогда. Твое сознание находится вне физического тела. Поймешь когда сдохнешь.
434 513800
>>13796
Смысл есть - я ж написал задачу: нужно измерить разность потенциалов между двуму объектами, связь между которыми возможна только информационная. Соответственно нужно каких-то 2 эталона чтоб возле каждого объекта измерить напряжение отноистельно эталона а затем передать эту информацию между ними.
435 513801
>>13797

>Поймешь когда сдохнешь


Ты то сам как это смог ощутить, родной?
436 513802
>>13800
Какая еще нахуй информационная связь?
В любом случае из-за относительности потенциала тебе нужен объект, относительно которого все измеряют. В электротехнике его называют нулем потенциалом.
437 513804
>>13801
Грибов поешь - узнаешь.
image.png180 Кб, 1023x409
438 513806
>>13802
Тебе нужно измерить напряжение между объектами А и Б, но напрямую элемтрически их связать нельзя.
439 513807
>>13748
Для того, чтобы объяснить, что такое N-мерное линейное пространство, сначала нужно объяснить, что такое вектор, объяснить векторную алгебру, ввести понятие "линейной зависимости"... да, это просто курс линейной алгебры для ВУЗов.
440 513808
>>13749

>Если f(x)=x+y


Тогда уж f(x,y).
441 513809
>>13807

>вектор


Причем в более общем смысле чем это преподают в школе. Это не просто блядь "стрелочка"!
442 513810
>>13806
А теперь скажи мне, почему эту задачу нельзя решить через ноль? Ноль, нейтральность - эталонный потенциал.
443 513811
>>13810
Как ты это на практике представляешь? Ноль на ситороне А не связан электрически с нулем на стороне Б. Как ты между этими нулями потенциал вычислишь?
444 513812
>>13797

>Твое сознание находится вне физического тела.


Можешь в таком случае ответить — что такое это внетелесное сознание? Потому что за способность любить, бояться, узнавать вещи, испытывать религиозные чувства отвечают вполне известные отделы мозга. За память тоже отвечает мозг — это доказывают тысячи склеротиков.
Так за что отвечает это непонятное внетелесное сознание? Какое это вообще сознание — без памяти, без эмоций, без мыслительного аппарата?
half head.jpg199 Кб, 1908x1146
445 513813
>>13812

>Потому что за способность любить, бояться, узнавать вещи, испытывать религиозные чувства отвечают вполне известные отделы мозга.


Как там в 19ом веке? Пикрилейтед отлично себя чувствует, память не потерял, курит травку и любит толстых девок https://www.youtube.com/watch?v=YYmfSbfA5iU
446 513814
>>13806
При такой изоляции потенциалы вообще будут самостоятельными, а их связь будет неопределенной. Ситуация примерно соответствует гальванической связи. При отсутствие ее потенциалы гуляют произвольным образом.
447 513815
>>13813
Ты юморишь так? Или всерьёз хочешь сказать, что в его голове отсутствуют значимые отделы мозга?

Вообще, у какого-нибудь Валуева форма черепа примерно такая же, лол.
448 513816
>>13814
В этом-то и вопрос - как измерить потенциал двух изолированных друг от друга тел.
Согласно определению потенциала - это отношение протонов к электронам, тоесть электрический заряд. Соответственно каждое тело должно иметь некий абсолютный заряд, не связанный с другими телами.
449 513817
Предлагаю топить за сознание как фундаментальное свойство материи, двухщелевой эксперимент, вот это всё.
Примерно как масса норовит скатится в ямки на скатерти ПВ
Так и сознание повышенным потенциалом накапливается в определёных локациях в силу специфических условий создаваемых присутсвием в данной области ПВ мозга/нервов/жизни/хоть какой-то самоорганизации, а в остальных местах размазано тонким слоем по довольно тупой неживой материи.
Я бы скозал вселенское сознание это "причинное" поле.
Чем выше концентрация причинно-следственных связей на единицу объёма тем выше уровень сознания.
При этом по этому полю в теории можно пускать и ловить волны, чем объясняется телепатия/внетелесный опыт/призраки и т.п.
Untitled.jpg91 Кб, 600x454
450 513818
>>13815
Лол у него отсутствует больше половины мозга, при этом он вообще не изменился. Или ты один из тех дурачков что искренне верят в миф "мы используем только 5% мозга" или ты прикалываешься.
451 513819
>>13779

> что загрузку сознания в том или ином виде изобретут в ближайшие 20-30 лет


Изобрели задолго до появления людей. Называется "воспитание детей" эта загрузка.
452 513820
В каком количестве здоровый человек может пить кипячёную морскую воду? Есть ли смысл её пить вообще в отсутствие пресной воды, или почки на вывод соли потратят больше воды, чем ты получил? Есть ли смысл её пить для восполнения солей и минералов?
453 513821
>>13817

>Предлагаю топить за сознание как фундаментальное свойство материи, двухщелевой эксперимент, вот это всё.


Какие у нас причины считать сознание фундаментальным? Кроме льстящего нам мнения о себе как о чём-то исключительном, конечно.

Мне самому сознание кажется удивительным, но такой же удивительной кажется мне и жизнь. Жизнь — фундаментальное свойство материи? Почему-то никто так не считает.

Потом, как быть с тем фактом, что сознание бывает очень разным? Возьмём классический пример: червя-нематоду C.Elegans с мозгом из сотни нейронов. Если мы признаём, что какое-то примитивное сознание у этого червя есть, оно сразу теряет ареол загадочности. Если же мы отказываем червю в любом сознании, следующим в очереди встаёт вопрос о сознании улитки. Тогда, рано или поздно, нам придётся от балды провести черту, сказав, например, что у пчелы сознания ещё нет, а у мыши-полёвки оно уже есть. Я саму попытку такого деления нахожу ошибочной.
454 513822
>>13820

>пить кипячёную морскую воду?


Нельзя, кипячение только дезинфицирует, что в принципе особо и не нужно, морская вода сама убивает большинство патогенов.
455 513823
>>13818
Ну вот видишь, поврежден только неокортекс, а там нейроны могут перестраиваться. И я не верю, что травма никак не отразилась на его памяти или умственных способностях.
456 513824
>>13817

>При этом по этому полю в теории можно пускать и ловить волны, чем объясняется телепатия/внетелесный опыт/призраки и т.п.


Чтобы начать объяснять все эти вещи, для начала их следует всё-таки зафиксировать.
457 513825
>>13815
Зачем кормишь тролля?
458 513826
>>13817

> чем объясняется телепатия/внетелесный опыт/призраки и т.п.


Я чай расплескал со смеху
459 513828
>>13820
Если нужна вода и есть возможность кпятить, собирай дистиллятор.
460 513831
>>13822

>Нельзя


Ни в каком количестве нельзя? Чёт шляпа какая-то получается.
461 513836
>>13816

>Согласно определению потенциала - это отношение протонов к электронам


Чего блядь?
Электромагнитный потенциал это вообще 4-вектор(тащемта линейная форма), которое отделяет ЭМ поле как таковое от других сущностей.
Потенциал самостоятельная величина и локально может быть ненулевой без носителей заряда. Собственно эм волны существуют, а это все равно что закрученный гиперспиралью 4-потенциал.
Заряд (точнее 4-ток) это, то как частицы взаимодействуют с эм полем. И это величина другого сорта, нежели чем потенциал.
462 513837
>>13821
Чего ты льстящее увидил? Яж говорю - свойство материи, и как пример привожу сраный электрон, куда там ему до гигантов мысли червей которых ты перечислил, а уже сознание.

>>13824
Как ты призрака зафиксируешь, синей изолентой? Он же сквозь стены ходит, что ему та изолента.

Короче сознание это такое базовое свойство/функция материи, без которого невозможно сколлапсировать волновую функцию.
Если бы его не было - вся вселенная так и осталась бы неопределённой вероятностной хуетой.
И чем выше уровень сознания тем точнее и ближе к пределу определяется импульс+координаты.
Таким образом вселенная естественным образом развивается в сторону повышения уровня сознания и снижения степени неопределённости. Результаты этого процесса называются смыслом.
"Смысл жизни" - делать вселенную всё олее настоящей.
Большой взрыв не был, он всё ещё в процессе и закончится он когда будет "ну всё понятно".
463 513840
>>13808
Бля, сорян, забыл
464 513842
>>13837
Твой пост нужно в шапку копировать, как "если вы из этих, то сразу пиздуйте нахуй",
465 513843
>>13836
Упрощенно и в данном контексте можно определить как отношение положительных и отрицательных зарядов, которые и определяют потенциал.
Вопрос был не в определении потенциала а в том как его измерить для изолированных объектов.
466 513845
>>13843

>как измерить потенциал изолированного объекта


абсолютное значение никак - его по идее вообще нет - просто сказади, что на бесконечности 0 и вот тебе.
можешь померить относительно бесконечности - постреливая вдаль заряженными частицами и смотря как они возвращаются, но это уже сложно назвать изолированной системой.
467 513847
>>13843
Упрощенно потенциал нельзя определить как заряд системы.
Если уж хочет связать электростатический потенциал с зарядом, то тебе придется использовать понятие электроемкости, а это уже более сложное понятие. И то, это только для статического случая.
Еще раз, потенциал это 4-вектор ебаный. В добавок ЭМ обладает калибровочной инвариантностью, а это значит нам не важен сам потенциал, а только скорость его изменения. Или поведение системы не поменяется, если есть внешний заряд.
И если мы имеем две изолированные системы, значение потенциалы между ними становится бессмысленным.
Или если есть изоляция, то одна система электромагнитно не взаимодействует с другой, и внутренние терки одной системы имеют нулевой пук для другой. Короче нельзя померить.
468 513849
>>13837
Да, я тоже читал Пенроуза. Он мужик умный, но тем не менее всё это очень похоже на сказку.
Ты, например, слишком по-бытовому понимаешь термин «наблюдение» из квантовой механики.
469 513852
>>13837
Волновая функция коллапсирует не из-за сознания, учи матчасть
470 513853
>>13837

> Яж говорю - свойство материи, и как пример привожу сраный электрон, куда там ему до гигантов мысли червей которых ты перечислил, а уже сознание.


А что нам, собственно, даёт такое определение сознания? Это ведь как сказать "каждый атом обладает фундаментальной примитивной жизнью".
471 513854
Не проще ли сказать, что атомы обладают способностью к выстраиванию систем, обладающих характеристиками, существенно отличающимися от внутреннего строения самих атомов?
Например, атомы могут складываться в воду, которая мокрая. Значит ли это, что каждый атом обладает фундаментальной мокротой?
На мой взгляд, фундаментальность мокроты не нужна точно так же, как фундаментальность сознания.
472 513855
>>13854
Если твою рэч разложить на атомы, билят, то даже атомы звука твоей рэчи будут говной вонять.
473 513857
>>13831
Пить можно в любом количестве, просто из-за большего содержания соли, для вывода этой соли нужно еще больше пресной воды, чем то количество, что ты выпил. Т.е. да, грубо говоря ты пьешь в минус.
474 513858
А как пьют рыбы и морские животные?
У них есть встроенные опреснители?
475 513859
>>13858
Да. Специальные железы, почки приспособленные под это. Дельфины вон вроде вообще не пьют, только едят.
476 513860
>>13672
Это ответ. Ты считаешь, что для выживания необходимо зрение, но летучие мыши чихают тебе в лицо ковидом.
Точно так же собака могла бы спросить "почему человек рождается с фиговым нюхом и пассивно лежит на спине вместо того, чтобы лезть к мамке под бок, как вы вообще выживаете"

Алсо, хотя ему ничего не грозит, зрение с первых месяцев используется для тренировки распознавательных отделов мозга и базового для человека навыка различения лиц. Щенки же учатся различать по запаху, а не по морде.
477 513862
>>13860

>базового для человека навыка различения лиц


Можно пруфца? На базовость навыка?
478 513863
>>13862
Пруфец в стайности человека.
479 513864
>>12263 (OP)
Вопрос не очень тупой, но не хочется создавать отдельный тред.
Есть ли книги, спекулирующие твердота рассуждений не важна, я посредственный гуманидаун о роли радиации в эволюционном процессе? Желательно не слишком сложно написанные. Пару раз наткнулся на упоминания экспериментов с генами и радиацией, и теперь зудит интерес, насколько вообще велик вклад фоновой радиации и ее вспышек в формирование видов.
480 513865
>>13863
А что выблдок решает? Не того выблядка просто выкинут взрослые, если так решат.
481 513868
>>13863
Нет там никакого пруфца.
Стайность подразумевает подавление индивидуализма в составе рода/семьи, и на этом буквально нахуй всё. Стайность - это как если бы каждую одиночную особь ебошили по голове доской во имя выживания вида. О развитии зрения это нихуя не говорит. Ещё раз попробуй, я терпелив.
482 513869
Где почитать объяснение того, как клетки собираются в такие сложные симбионтные формы, как организм человека, ну или в более простой организм? Как происходит "договор" между ними, по какой логике они соединяются и как возможно собираться в огромную колонию, вроде нашего тела и чем осуществляется управление этими сложными взаимодействиями? Централизованное оно или децентрализованное, как эти отдельные колонии клеток вообще умудряются действовать как единое целое? Как это объясняют эволюционисты?
483 513870
>>13864
сходу не нашел текста, но вот тебе тезис. На превращение человека из волосатрй и сильной в лысую и слабую обезьяну мог повлиять естественный радиоактивный реактор в Осло(Габона, центральная Африка). Сильная обезьяна стала ущербно-лысой и выжили только умные.
9 Лет Дом Убивал Людей Радиацией.mp414,3 Мб, mp4,
1280x720, 3:57
484 513872
>>13870
Я бы лично хотел облучить всех эволюционистов-атеистов, их жен и детей той дозой, что они сами назовут (но не меньше определенного порога в скажем 2 Зиверта). Вот и посмотрим, станут ли они от этого умнее или нет.
485 513873
>>13868
Не он. Вот тебе 4 пруфца, осилишь - продолжу беседу с тобой, нет - ты не стоишь таких усилий.
486 513874
>>13869

>как клетки собираются


Между человеком и одноклеточными были промежуточные формы. И немного времени.
А теперь собираются довольно сложным путем, но лучше там на червячках пойми, прежде чем к человекам переходить.

>чем осуществляется управление этими сложными взаимодействиями


Ныне куча путей, специальные вещества, электрические импульсы, сложные системы доставки, клетки регуляторы, команды для самоуничтожения. Изначально химией в основном.

>Централизованное оно или децентрализованное


И так и так, изначально децентрализованно, но потом оказалось выгодно иметь центры управления и специализированные органы.

> Как это объясняют эволюционисты?


Ну вот сам объясни, чем дерево может быть выгоднее одноклеточной водоросли. Что оно может, а водоросль не может?
487 513875
>>13873
Показал младенцу, он орёт и ничего не говорит о том, что распознал лица. На парашу пиздуй вместе с первоначальным постером, долбитесь там себе в дупла.
488 513876
>>13875
Этим младенцем был конь ты? Ок, на этом и закончим, раз ты даже такого простого примера не понял.
489 513877
>>13876
Протип: туземцы Амазонии не видят в фотографиях людей людей, они видят просто бумажки.
Ты серишь, понимаешь, серишь. Выдаёшь желаемое за факт, и даже когда тебя ткнули в то, что это нихуя не факт и это надо доказывать, ты продолжаешь серить.
490 513878
>>13877
Хули тут доказывать, кроме самоочевидности факта? На 3 фотках хуета даже не близкая к человеку, на последней пикассо стайл перевернутые части лица. И везде люди видят лица, палка-палка-огуречик, сука снова распознали лицо. Пока оно не перевернуто. Еще раз, если ты не можешь осознать очевидного и хочешь доказательств - ебошь в гугл и обдаказывайся, еще бы спросил почему 2+2 = 4 с доказательством.
image.png340 Кб, 637x476
491 513879
>>13878
Ты с кем и о чём разговариваешь вообще? Ты понимаешь, где ты находишься, какой сейчас год, как тебя зовут?

Я напоминаю, что речь идёт о младенцах и их инстинкте вцепляться глазами в лицо, ведь они такие социальные.

На самом же деле, маленькие дети не смотрят на лица. Они их не интересуют. И только когда ребёнок начинает ползать и что-то делать сам, он начинает пытаться распознать эмоциональное состояние родителя, потому что если он не угадает, то получит пизды. Причина - следствие.
492 513880
>>13879

>>базового для человека навыка различения лиц


>Можно пруфца? На базовость навыка?


Пруфец на пикрилах.

>идёт о младенцах и их инстинкте вцепляться глазами в лицо, ведь они такие социальные


Не идет, ты хотел пруфца на базовость навыка. Вот оно. Личинусов без меня препарируйте, мне похуй на какой неделе они взгляд фокусируют, а на какой лицо начинают отличать. Принцип анон тебе правильно передал, предкам важно было стереозрение больше, чем нюх и слух, для прыганья на ветки и распознавания силуэтов и лиц для своих и внешних опасностей. Копытным вон важно сразу бегат начать, так что они встают на ноги чуть ли не сразу как обсохнут после рождения.
493 513881
>>13872
тебя видать в утробе до сверхчеловека облучили, что ты так преисполнен.
494 513883
>>13880
Давай попробуем ещё раз. Ты, похоже, не понимаешь, как работает наука, верно?
Статью мне принеси, где нейрофизиолог говорит то, что ты говоришь. Если это самоочевидный факт - мне нужна статья про самоочевидный факт. Не от дауна из даунотреда, а с референсами и опытным подтверждением. Ты всё смог прочитать, что я написал? Молодец. Ещё раз принесёшь мне смишные картинки, и я покажу тебе, как выглядит твой носовой хрящик.
495 513884
>>13883
Душный ты. Тебе надо - ты ищи. Не твоя личная.
496 513885
>>13884
Что и требовалось.
Такие как ты орали, что Земля плоская, потому что это самоочевидный факт.
497 513886
>>13885
Это не значит, что подтверждений нет, это значит что ты будешь сейчас ебать мозг и просить пруфов на каждый чих. Ебись сам, я тебе точно не буду тащить статьи чтобы ты тут РЯЯ вопил.
498 513887
>>13886
Ебаться тут решительно не с чем. Ты сморозил хуйню, ты обосрался её доказать и был озалуплен. Фин.

>новорожденные могут сразу ходить!


>пруфы?


>встань и иди, это же очевидный факт

499 513889
>>13887
Ты на картинках лица видел, даун? А там их нет. Иди озалупливай мамку своим мозгоебством.
500 513890
>>13889
Но я же не новорожденный, няша. Судить по моему поведению (тем более предполагаемому тобой, дебилом) об априорном состоянии человеческой психики - это не очень умная затея, тебе не кажется?
501 513891
>>13890
Ты просто слепой, да. Даже первоначальный анон не говорил, что новорожденный распознает лица, но ты приебался с этим ко мне. Базовость навыка опровергнуть можешь?
502 513896
Что изучать, чтобы стать аэрокосмическим инженером и работать на Илона Маска?
image.png327 Кб, 608x375
503 513898
>>13891

>Базовость навыка опровергнуть можешь?


Разумеется. Но чтобы что-то опровергнуть, надо сначала что-то доказать. Ты доказал своё утверждение, няша?
504 513900
>>13898
См. пики с розетками и рыбами.
505 513903
>>13900
См. мой ответ.
506 513904
>>13903
Да, ок, ты прав, я не готов продолжать. Ты хуже моей бывшей в плане мозгоебства.
507 513905
>>13904
Поздравляю, ты тупее бабы.
EarthfromISS1.jpg117 Кб, 900x599
508 513907
>>13905

>>>>>Почему земля круглая?


>>>>См пик


>>>Нет, ты статью астронома покажи!


>>На пике видно и так


>Ты тупее бабы, РЯЯЯ


Ок.
509 513908
>>13907
На твоём пике она плоская, зая.
Кстати, что я говорил насчёт смишных картиночек?
image.png189 Кб, 314x274
510 513911
>>13907
Ну тащемта в этом примере ты нихуя не доказываешь. Или мой пик доказывает, что Земля - куб.
Мимо
511 513912
>>13911
У него вообще вопрос поставлен как

>Почему земля круглая


>Почему


Ответ будет "из-за воздействия гравитации". Но поскольку это хлебушек, он даже на собственный гринтекст ответить не может, не обосравшись.
512 513913
>>13912
Я это списал на невнимательность. Сам, бывает, думаю одно, пишу другое, потому не придрался
513 513914
>>13877
Пруф на туземцев? Интересно почитать.
Я вот замечал что даже кошки / собаки способны распознавать плоское изображение. Но это отдельная проблема - плоские изображения, итт речь про распознавание лиц а не про распознавание плоских изображений лиц.
514 513915
>>13914
Искать лень, отрывки из антропологической энциклопедии читал.
Они ещё и свой голос на магнитофонной записи не узнают. Просто отказываются и выдумывают разнообразные агнозии.
Заставляет задуматься, а действительно ли мы способны изменить своё мировоззрение, столкнувшись перед неоспоримыми фактами, которые ему противоречат.
515 513921
>>13915

> а действительно ли мы способны изменить своё мировоззрение, столкнувшись перед неоспоримыми фактами, которые ему противоречат.



нет, именно поэтому ученые и пытаются ебошить максимально непредвзято и максимально абстрактно, чтоб мировозрение не мешало.

как пример можно привести работы эйнштейна в квантмехе.
516 513925
ИТТ злой кун спорит с термином "базовый навык" ошибочно трактуя его как "врождённую способность" - отсюда идёт невтемные ссылки на младенцев.

Само слово "навык" подразумевает обучение, в таком случае "базовый" читается не как врождённый, потому что это оксюморон, а как "низкоуровневый", "основополагающий" и т.п.

Далее по делу идёт ссылка на хуёвый нюх и приоритет на бинокулярное зрение что бы хвататься за ветки - это годный ответ на изначальный вопрос.
Ссылка на оленей встающих сразу тоже гуд (у них кстати тоже глаза открыты сразу)
От себя добавлю что собаки/кошки/лисы/волки живут и размножаются в норах, там темно и смысла торопиться с глазами особо нет.
При этом вышеупомянутые звери - в (том числе) ночные хищники, и глаза у них более чувствительные, так что возможно это тоже причина почему их долго не распаковывают, так как им нужно время что бы дозреть/адаптироваться.

Фотки с глазастыми розетками, краном, домом, итп не в тему.

Всё, можно разойтись по углам.
517 513931
Не могу найти видео.
Препод рассказывает студентам зачем учить физику: "Жизнь ставит перед нами проблемы. Физика как никакая другая наука учит решать такие проблемы. Либо вы их решаете, либо скатываетесь на обочину жизни".

Запись всратая с первой парты.

Помнит кто такое?
518 513932
>>13931
Это лекция по ТРИЗУ от одного из создателей.
519 513933
Поясните, как работает земля в электротехнике. Вот чтобы ток шёл по цепи, должен быть контур. Но почему достаточно провести один провод между устройствами, а вторые концы воткнуть в землю, и якобы сложится контур? Как электроны понимают куда им бежать под землёй?
520 513934
>>13933
Электроны они массовка. Там вся фишка в электромагнитном поле.
521 513935
>>13934
Как начать изучать электротехнику и не получить 220 прямо в сердце?

Бати нет, деда нет, но есть желание научиться паять.
522 513936
>>13935
Ну 220 в сердце это не страшно. Даже 10000 не страшно. Ведь убивает сила тока а не напряжение.
По электротехнике не могу ничего подсказать. Начни с теории, скачай учебники, потом на бумаге примеры посмотри. Есть программы, которые позволяют моделировать разные цепочки, прежде чем ты их спаяешь. Удобно.
sage Kehel 523 513938
Сап, науч, необходимы книги по теории поля. Если у кого-нибудь есть таковые (имею ввиду толковые, с пояснениями, а не онли на научке), прошу дать автора и год издания. Дополнительные материалы приветствуются. Заранее благодарен.
524 513939
>>13938
А ты знаешь что-то о гармонических колебаниях и электромагнитных волнах?
525 513940
Из каких элементов состоит механизм возникновения желания размножаться у вас, людей? Замечаю, что любой человек стремится необоснованно копировать свою ДНК при первой возможности.
526 513945
>>13940

>необоснованно


Доказать сможешь?
sage Kehel 527 513947
>>13939
Да, но, конечно, я не машина и мне свойственно, как и любому другому человеку, забывать. Если есть материал, прошу Вас поделиться. Считаю, любое знание лишним не будет. А повторить базис - дело благое.
528 513950
>>13947
Конкретно не скажу, извиняюсь, но возможно поможет:
https://dxdy.ru/topic121655.html
529 513960
>>13940
Все кто не желал - вымерли, кроме тебя конечно, ты то уж точно не вымрешь.
530 513986
>>13925

>отсюда идёт невтемные ссылки на младенцев.


Мы начали с обсуждения младенцев, если вдруг чо.
doom-simulation-political-cartoon.png69 Кб, 1500x500
531 513992
Внезапно настал возраст почемучки. В 25 лет я понял, что знаю о мире ровно нихуя, кроме того факта, что деревья как-то производят кислород.

Ещё не слишком поздно наработать и развить в себе мышление ученого? Если не поздно, то как сделать? Что читать, смотреть, с кем общаться?
532 514001
>>13960
Умерли все те, кто не жив сейчас. Ты же не переносишь сознание в тело своего пиздюка, продлевая тем самым своё существование. Ты в той же мере свой сын, в которой я твой сын (wut?).
>>13945

>Доказать сможешь?


Да, нет жизни — нет проблем.
533 514005
>>13992
Для вдохновения могу посоветовать почитать Фейнмана
timeline145495301000024.jpg28 Кб, 345x612
534 514008
>>13932
Затрахался рыть ютуб.
Не помнишь конкретнее, а?
535 514009
>>13992
Деревья не производят кислород.

Сколько дерево за жизнь произведёт кислорода - столько же оно и поглотит при гниении/горении.

Намного более важная задача деревьев - связывание углекислого газа.
Если мы сегодня же сожжём все деревья, уровень кислорода понизится незначительно. А вот углекислый газ бахнет так, что нам придёт пизда.

Основная масса кислорода порождена бактериями оч давно.
Понемногу кислород, вроде, может генерироваться на краю атмосферы из-за распада озона под действием Солнца, но это не точно.

Ближе к вопросу.
Что делать - идти учиться туда, где ещё учат. Наука на 90% состоит из нудных вычислений и осознания своей никчёмности. Простого пути нет.

Без дураков советую "Занимательную Х" от Перельмана, а также все научпопы СССР, начиная с Центрнаучфильма.
https://www.youtube.com/watch?v=ECvsD4b0JlU

Крепка была советская наука)
536 514011
>>14009

>Крепка была советская наука)


Весьма сомнительное утверждение.
537 514012
>>14009

>Деревья не производят кислород.


Ох уж это острие науки. Любые зеленые растения при облучении ультрафиолетом, доступе к воде и углекислому газу могут CO2+H2O->O2+C6H12O6, вопрос в эффективности световой фазы (больше инсоляция - больше производит кислорода, дольше темновой период - больше СО2). Основную массу кислорода производят водоросли, деревья что-то около 10%, так что их рассматривать, как основных производителей О2 в первую очередь глупо, видов фотосинтеза основных аж 3 штуки: C3, C4, CAM, разные по эффективности темновой фазы (от этого зависит где могут жить, следовательно как много УФ поглощать), подробности в гугле.

>Намного более важная задача деревьев - связывание углекислого газа.


Ну хоть тут не объебался. Наряду с океанами, да.

>Основная масса кислорода порождена бактериями оч давно.


Это называтеся кислородная катастрофа и ты ее понял неправильно. Это не запас кислорода, а резкое изменение состава атмосферы, от которой мы получили то, что имеем тут.
538 514024
Ребят, еле вас нашел через каталог. У меня встал вопрос по данной задаче:
До ближайшей звезды Альфа Центавра примерно 4 световых года. Мы отправляем к ней супер звездолёт, с экипажем, способным мгновенно развить скорость света. Звездолёт финиширует в районе звезды через 4 года. Мы состаримся на земле тоже на 4 года. Сколько времени проведёт экипаж внутри звездолёта?
Какой правильный ответ?
539 514027
>>14024

>мгновенно развить скорость света


>Какой правильный ответ?


Звездолет на ноль поделило.
540 514030
>>13940

>желания размножаться


У людей нет инстинкта размножения, тк нет периода гона. Желание размножаться культурно обусловлено.
541 514033
Как работает слух? Как так получается, что звуки не сливаются в одну картину и можно отследить несколько источников шума?
Почему запахи сливаются в какой-то один доминирующий, а другие почти не слышно?
542 514053
>>14012
Спасибо)
543 514076
Тоже про размножение вопрос.
Вот собаки знают, как трахаться, даже если их с детства держали дома и они не видели чужих спариваний.
А люди не знают, если их не научить - почему?
544 514086
Посоны, а реально ли сделать ядерный ракетный двигатель на искусственных изотопах лёгких ядер? Сделать какой нибудь гелий-7 в рот его ебать, потом подать его в плазменный двигатель, где магнитная ловушка плюс температура заставят его усиленно делиться.
545 514087
Топливо из изотопов легкий ядер я так понял должно обладать низкой критической деления, попадая в сопло с магнитной ловушкой и облучением будет получаться критическая масса топлива и космолёт полетит на миниатюрном ядерном взрыве в дупле.
546 514092
>>14087
>>14086
Затраты энергии на магнитную ловушку и поддержание плазмы расчитал? Оно у тебя и на борту как минимум ядерный реактор потребует для этого потребует, а это +масса+масса топливо -> увеличение затрачиваемой энергии на полёт.
547 514094
>>14086
>>14087
А ничего, что все после гелия-4 само распадается за хуеллиардную долю секунды? И в чем смысл сего действия? Импульсный ядерный/термоядерный интереснее выглядит. Только укачивает поди.
548 514111
>>12263 (OP)
Возможен ли swampman?
549 514113
>>14094

>Импульсный ядерный


Если на один импульс при сильном сжатии будет приходится 100 миллиграмм вещества, а импульсы будут пару раз в секунду - это будет как ДВС
изображение.png7 Кб, 365x286
550 514114
Вам предоставлена следующая информация. Заемщики распределены по возрастам следующим образом: При этом нет заемщиков моложе 23 лет и старше 50 лет. Среди заемщиков

Вероятность того, что наугад взятый заемщик женщина 40-45 лет равна
А) 15%
Б) 8%
В) 3%
Г) нет верного ответа

Пожалуйста напишите как решать подобные задачи по статистике с вероятностью.
В идеале название способа и формулу для именно этого случая.
551 514132
Что дало решение Гипотезы Пуанкаре???
552 514143
>>12263 (OP)
Как получается, что мы не можем глазами увидеть свой нос??? Только его кончик, когда скосим глаза. А потом он сразу исчезает и вообще теряешь на пару секунд мысль, что ты смотрел на свой нос.
553 514153
>>14143
у тебя боковое зрение что ли не работает?
Всегда вижу свой нос без скашивания. Попробу раздувать/шевелить ноздрями - боковое зрение лучше работает на динамическую картинку.
554 514165
Я точно помню, на лекциях по физике нам показывали слайды с чем-то вроде поатомной сборки геометрических фигур. Как эта штука называется?
555 514173
>>14143
Всегда прекрасно вижу свой нос. Но он у меня очень большой.
556 514178
>>14173
Таки да
557 514213
>>14165
Нанотехнологии.
558 514214
>>14033
Не сливаются, это всё от работы мозга зависит, тренировки, образовавшихся нужных нейронных связей и алгоритмов обработки которые они представляют.

Мне больше про свет интересно.
Как глаз может видет тусклые звёздочки когда прямо в него хуярит фонарь который в миллиарды раз ярче точек на небе.
559 514215
>>14024
Скорость света недостижима, но при бесконечном приближении к ней для экипажа корабля пройдёт бесконечно мало времени.
560 514228
Я придумал альтернативу крионике: запускаем космический корабль с собой на борту к ближайшей чёрной дыре и огибаем её по малому радиусу. Получается как в Интерстелларе (или даже как в сериале «Андромеда»): за время манёвра для внешего наблюдателя проходят сотни лет, а для команды корабля время почти не движется. Возвращаемся на Землю в технологический рай.
561 514244
>>14228

>а для команды корабля время почти не движется


ближайшей чёрной дыре


Пока ты к ней долетишь, технологический рай или пиздец и так наступит.
562 514259
Почему, когда сыплешь перец в молоко, то оно резко расползается как бы?
563 514260
>>13870
>>13864
Статью про реактор прочитал, но дальше мыслей, вроде всякого палеоконтакта и иже с ним, ничего эта статья не пробуждает. Насколько всё-таки и каким образом радиация ответственна за формирование видов, в особенности человека. У кого-то есть информация на этот счёт, желательно в книжной и общедоступной форме? Ну или хотя бы статьи на каком-нибудь лессвронг/вики?
15966200641260.jpg42 Кб, 399x534
564 514262
У разума есть ограничение на размер носителя в виде головного мозга? Может ли мышка, например, обладать разумом не уступающим моему разуму (только без петросянства), если её мозг сильно усложнится и обзаведется неокортексом? Разумеется сохранив свои мышиные габариты.
565 514272
>>14262
Удваиваю вопрос. Наверняка, важен размер мозга, наравне со сложностью его структуры, но хотелось бы узнать от сведущих, почему это так.
566 514279
>>14272
Я тот анон, что задал вопрос. Даже мне ОЧЕВИДНО, что бОльшая масса нейронной ткани всегда равно больший интеллект в пределах одного организма. Вопрос в том, можно ли вообще сотворить разумную мышку, если её мозг будет сложнее моего. Я почти ничего не знаю про принципы работы мозга, почти ничего про типы тканей и как интерпретировать эти волны электрической активности, испускаемые мозгом. Есть множество прецедентов как очень умные люди теряли половину своего интеллекта в результате ЧМТ, при этом чисто на глазок, через обследование МРТ, их мозг ну совсем не получил повреждений. Так что на изначальный вопрос может ответить только доктор нейробиологии. А учитывая, что научному сообществу известно маловато информации о мозге и принципах его работы, то даже доктор наук может лишь примерно прикидывать, что мозг человека мог бы быть и лучше, если бы "то-то да сё-то".
Однозначно только то, что толщина неокортекса людей приблизительно 3-4мм, а толщина неокортекса у негров как правило меньше (по информации из запрещенных учебников от мировых ученых 70-90х годов), и мы естесственно видим более низкий интеллект, чем у европеоида. Я не европеоид, если что, я финноугр, так что никакого "расизма" (что бы эти либеральные мракобесы не вкладывали в это слово).
567 514282
У половой клетки (например сперматозоида) и других клеток организма (например клетки кожи) одинаковое ДНК или нет?
145301714721417203.jpg121 Кб, 570x319
568 514309
>>14279
Я тот анон, что удвоил. Второй абзац порадовал. Смотреть древний мем на пике. Видимо все дело именно в структуре, если ЧМТ не оставило видимых повреждений, значит она похерила устоявшиеся связи, на которых все держалось, а без них вся структура ломается. Всё-таки особенности софтваре в этом всем не так хорошо изучены, как хардваре. Поэтому психологи ебутся с недоказуемыми абстракциями и проекциями желаемого на действительное, а бихевиористы и нейробиологи с тупым мясом без центрального вектора, которое хер пойми чем, кроме примитивных процессов вроде поддержания гомеостаза управляется. Инбифо: никаких инстинктов нет. Личность это иллюзия, а думающая и делающая части личности это две никак не связанные инстанции.
569 514320
>>13860

> Ты считаешь, что для выживания необходимо зрение, но летучие мыши чихают тебе в лицо ковидом.


Боже какой даун.

> Точно так же собака могла бы спросить "почему человек рождается с фиговым нюхом и пассивно лежит на спине вместо того, чтобы лезть к мамке под бок, как вы вообще выживаете"


Тут у меня вопросов не возникает, а только у тебя дауна с развитием собаки.

> Алсо, хотя ему ничего не грозит, зрение с первых месяцев используется для тренировки распознавательных отделов мозга и базового для человека навыка различения лиц.


Уже лучше, но вопросы все равно остаются.
570 514323
>>14262
Советую лекции о мозге в серии "Научный тык".

Большую часть лекций просрало время, к сожалению.
Если у кого с л у ч а й н о есть архив Страницы42 - я был бы счастлив забрать.
571 514333
Какие щас в науке доминируют взгляды на механизм эволюции? Приобретенные в течение жизни признаки не наследуются или наследуется хоть что-то приобретенное? Слышал про эксперимент где мыши начинали по-особому относиться к запаху растения, после того как их родителя вместе с этим запахом мучали.
https://www.bbc.com/russian/science/2013/12/131201_genetic_intergenerational_memory
200.gif1,1 Мб, 200x200
572 514359
Учёные до сих пор не научились конструировать, например, протеины на молекулярном уровне? Или есть какие-то успехи?
573 514364
>>14359
Твой вопрос непонятен. Что значит "на молекулярном уровне". Ты их и сам сейчас конструируешь. А как еще блять их можно конструировать если не на молекулярном?
574 514370
>>14364
Я не могу сконтруировать такой протеин, какой сознательно захочу сделать. То есть вот существуют программы для конструирования 3D-моделей молекул и т.п., но сами эти соединения поатомно собрать нельзя, такой технологии нет?
575 514374
Что такое время, почему нельзя вернуться в прошлое, почему оно вообще идёт. Почему только вперёд. Как со стороны выглядела бы наша вселенная?
15969532564130.jpg213 Кб, 927x674
576 514375
577 514379
>>14370
Почему же, есть. Причем можно тупо синтезировать "там такая робо-рука просто смешивает по-очереди аминокислоты и реагенты, которые сшивают их", а можно сделать сначала кусок ДНК, который копирует этот белок, а потом сделать какую-нибудь бактерию с этим участком, которая будет тебе этим белком срать.
578 514380
Такие дела, ананасы, завтра первый день на работе, в теме я вообще не шарю, велели почитать про светодиоды и как их проверяют - без какой-либо конкретики.
Посоветуйте годной инфы по теме или проводите в соответствующий раздел/тред, пожалуйста.

Можете ещё краткий курс школьной физики посоветовать.
579 514388
>>14379

>а можно сделать сначала кусок ДНК, который копирует этот белок, а потом сделать какую-нибудь бактерию с этим участком, которая будет тебе этим белком срать.


Мне это казалось чистой фантастикой. Как такое гуглить?
580 514398
>>14380
А чё за проверки?
Глян на ютубе канал берримор, дедок прикольный

Ещё один видос классные видел, про замер кпд, если захочешь-найду.

А вообще у светодиодов вольт-амперная характеристика и световой поток, плюс цвет и спектра свечения, полность этого спектра, если это важно.
581 514449
Перекат

>>514446 (OP)
Перекат

>>514446 (OP)

Перекат

>>514446 (OP)
582 516017
>>12263 (OP)
Как птицы и другие твари передают прлучают/передают знания о постройке своего жилья и добыче пищи? Тупо научением? Или есть некие базовые программы, если да, то где и как они хранятся?
583 529951
Итак, есть вопрос по математике.
Есть два фактора влияющих на результат. Первые два фактора можно выразить уравнениями. Надо предсказать результат
Например, имеем мощность и вес автомобиля, как первые 2 фактора, которые выражены уравнениями. Нужно составить такую систему уравнений, что бы предсказать скорость машины. Надо составить что-то универсальное, что бы подставлять любые значения для конкретных параметров
Как такое сделать?
Тред утонул или удален.
Это копия, сохраненная 27 апреля 2021 года.

Скачать тред: только с превью, с превью и прикрепленными файлами.
Второй вариант может долго скачиваться. Файлы будут только в живых или недавно утонувших тредах. Подробнее

Если вам полезен архив М.Двача, пожертвуйте на оплату сервера.
« /sci/В начало тредаВеб-версияНастройки
/a//b//mu//s//vg/Все доски